1st series [1] [2] [3] [4] [5] [6] [7] [8] [9] [10] [11] [12] [13] [14] [15] [16] [17] [18] [19] [20] [21] [22] [23] [24]  2nd series [1] [2] [3] [4] [5] [6] [7] [8] [9] [10] [11] [12] [13] [14] [15] [16] [17] [18] [19] [20] [21] [22] [23] [24] [25] [26] [27] [28] [29] [30] [31] [32] [33] [34] [35] [36] [37] [38] [39] [40] [41] [42] [43] [44] [45] [46] [47] [48] [49]

  View the latest questions and answers at askaphilosopher.org

Ask a Philosopher: Questions and Answers 29 (2nd series)

When referring to an answer on this page, please quote the page number followed by the answer number. The first answer on this page is 29/1.

(1) Helier asked:

Is it true that in science 'theoretical' means 'non-empirical'? If so, are theoretical entities radically imperceptible? That is, although we can perceive the effects of theoretical entities, we can never perceive the entities themselves. For example, theoretical temperature is average kinetic energy of molecules, which we cannot perceive, but we can perceive its effects as thermometer readings and sensations of hot and cold; or mass is imperceptible but we can perceive its effects as forces of weight and inertia.

---

Wow I just submitted a paper on this issue... but unfortunately that makes answering this very difficult, not easier. Um... no, "theoretical" does not mean "non-empirical". You can test for characteristics of, say, electrons even though we'll never see one. Or better, consider "potential energy". Once that's observed in any way, it's no longer potential, right? And your example of mass is a pretty good one also, almost as good as something which must disappear when it's disturbed... like potential energy. And both of those are empirical in the sense that they can be measured with physical instrumentation, tested for, etc., although those observations and tests must be indirect, i.e., actually of other entities from which we infer the existence and characteristics of the one being measured; and in the sense that they are affected by changes in other physical entities (which really amounts to the same thing). But the question of what is "imperceptible" is not an easy one. By the naked, unaided eye? With a microscope or telescope? Indirectly, as with an x-ray image? You see what I mean? Basically, that leaves as clear-cut cases just the entities which disappear or change into some other entity when they are disturbed, like potential energy, or entities which simply cannot be observed except indirectly, like mass. Notice that this also involves mental qualities. Is anger observable? Yes, but only by the person experiencing it.

So the upshot is that there are some entities which are indeed empirical in the above sense, yet which are "directly" - a term that must be carefully clarified - unobservable. And, in fact, what I'm arguing is that (some, at least) mental entities are also of this type, indeed, that they have an advantage over entities normally considered "physical" yet intrinsically directly unobservable, like mass: that they are directly observable by one person.

But there's no fundamental problem here that I can see... I mean, so what if something is only indirectly observable? If we get consistent measurements (repeatable, consensually verified, validated...), which are consistently affected by other physical entities, then we make inferences as to the characteristics of the unobservable entity and go from there, just as we do with observable entities which we cannot see clearly, completely, or of which we have not yet measured some aspect.

Steven Ravett Brown

back

(2) Nickson asked:

Do modern people hold to mythological, superstitious and syncretistic beliefs and practices? If not, why not? If so, what are those beliefs and practices?

---

You've got to be kidding. Go look at some of these:

Alper, M. The "God" Part of the Brain: A Scientific Interpretation of Human Spirituality and God. Brooklyn, NY: Rogue Press, 2001.

Barrett, J.L., R.A. Richert, and A. Driesenga. "God's Beliefs Versus Mother's: The Development of Nonhuman Agent Concepts." Child Development 72, no. 1 (2001): 50-65.

Davies, M., and M. Coltheart. "Introduction: Pathologies of Belief." Mind & Language 15, no. 1 (2000): 1-46.

Giovannoli, J. The Biology of Belief: How Our Biology Biases Our Beliefs and Perceptions: Rosetta Press, Inc., 2000.

Hines, T. Pseudoscience and the Paranormal: A Critical Examination of the Evidence. Buffalo, NY: Prometheus Books, 1988.

Sagan, C. The Demon-Haunted World: Science as a Candle in the Dark. New York, NY: Random House, Inc., 1996.

Schick, T., Jr., and L. Vaughn. How to Think About Weird Things: Critical Thinking for a New Age. Mountain View, CA: Mayfield Publishing Company, 1995.

Shermer, M. Why People Believe Weird Things: Pseudoscience, Superstition, and Other Confusions of Our Time. New York: W. H. Freeman and Co., 1997.

Young, A.W. "Wondrous Strange: The Neuropsychology of Abnormal Beliefs." Mind & Language 15, no. 1 (2000): 47-73.

Steven Ravett Brown

back

(3) Helier asked:

I am having trouble with secondary qualities, which are manufactured in the brain after receipt of digital signals from the sense organs. For example, if I see a green leaf, I know that chlorophyll molecules in the leaf transmit electromagnetic radiation of a frequency such as to produce a sensation of green in my brain. The problem is that all the empirical objects that I perceive are structures of secondary qualities, and these are all outside my head. So where are secondary qualities, inside my head, or outside?

---

Oboy. Well, first, you're first sentence is wrong and/or unclear on a number of points. "Secondary qualities" is a category employed in certain schools of philosophy, ok... but one can talk about both primary and secondary sensory qualities, and you seem to regard all sensory qualities as secondary. That's another way to classify things, yes, but it leads, in my opinion, to more problems than it's worth. In fact, the classification into "primary" and "secondary" sensory qualities is a mess, given the last 75 years or so work in cognition.

Next, the signals, if you want to use that term (and I hate it), that one "receives" (Look, the whole "sending" and "receiving" thing implies that the "sender" and "receiver" are trying to communicate. Is a rock trying to communicate with you? Speaking this way leads to more problems than it's worth. And yes, I know it's common these days. Unfortunately.) are not digital in any sense. Yes, yes, I know about "all-or-nothing" neural discharges. What's doing the sensory transduction, however, viz., the sensory interface (e.g., the retina, cochlea, etc.), employs, roughly speaking, an analog frequency code (I'm neglecting burst coding and parallel phase coding, and a few others... all of which are also analog).

I guess I'll pass over the problems inherent in calling it "manufacturing". As for "in" the brain... yes... sort of... good enough, I suppose (although there's a whole school of contemporary phenomenologically- and cognitive-inspired epistemology which would start screaming about the "body" here).

All the empirical objects you perceive are, let us say, outside your head. That's not strictly true, but let's take it as true for the sake of discussion. Now, the point here is that you're claiming that what you're terming "secondary qualities", like sensations of color, are "in" the brain, and that this is odd, given that the actual objects are "outside" the brain.

Well, first, there's yet another school of philosophy which will start throwing things at you here, and stoutly maintain that things like color are in fact "outside". This externalism vs. internalism debate is about... um... 3000 years old? However, in my opinion, they're simply wrong, given the last century or so of research into cognition, sensation, etc. That's way I'm so empirically oriented when I answer questions about the mind here... but anyway, ok, colors are inside, in some meaning of "inside" which I'm not going to try to clarify here.

But that still leaves this little problem. What's the difference between "feeling that" or "sensing that" or "apperceiving that" colors are "out" in the world, and knowing that colors are out in the world? Just because we assign colors to objects that we "feel", in some sense, are outside, doesn't mean that they are outside... it only means that we think or feel, or whatever term you want to use here (and that depends on the school you're coming from...) outside. You can have a perfectly consistent position to the effect that sensory qualities like green are characteristics of objects that we assign, and feel, and think, are objective, yet are in fact internal constructs which are set in motion, so to speak, by disturbances of various sorts of our sensory organs. This is, by the way, pretty much straight from Kant, from the 1700s.

Steven Ravett Brown

back

(4) Ramon asked:

Ok here is the situation...I have studied many philosophical problems, specially that of the analytic tradition but I still haven't read any major text of a certain philosopher, lets say... Wittgenstein, I always get a basic (general idea) of the book but never really gotten to read it all. The thing is that im currently writing some of my ideas and propositions for further elaboration in the future. It is my believe that reading other philosopher's ideas dulls the creativity, one gets contaminated by other peoples thought and originality goes out the window. But I am desperate for advice, because I don't know if to read some major philosophical works or to go on as I was. Should I read more philosophy?

PS: My work is based on the idea of concepts, conceptual analysis, ontology and philosophy of language.

---

Do your own thinking first, definitely. Get a job like that of a lighthouse keeper, where you are needed there all the time but don't have much to do. (Unfortunately there are no lighthouse keepers any more, but there are those kinds of jobs - usually isolated ones.) Then when you're satisfied with your system of thought you can study philosophy - preferably history of philosophy and great books in philosophy. You will find that some of your ideas are naive, many of them have been thought of before, and some (if you're any good) are original to the subject. But at the very least you will have learnt to think creatively, uncontaminated by other people's thought. Good luck!

Helier Robinson

back

(5) Fred asked:

Am I just a chemical brain? For years I have been fighting within myself to accept something and I am still fighting over this. Everything we do, say or think comes from our brain of chemicals so what am I supposed to accept, that my whole existence to reality is just a bunch of cells, synapses and neurons??

I would really love to have someone ease my suffering on this matter as it has driven to wits end. I have been told there are no answers to my questions but I cannot accept that I am just this chemical brain and it creates my thoughts and is my consciousness... Please help!

and Morris asked:

The question. Inasmuch as our mind is enclosed in our physical body, isn't all of us just a matter of the arrangement of the atoms/molecules within?

I add the below solely as an attachment.

Or differently written, we are what our tissues, DNA, blood, organs, et al, are? Open us up, there's no ghost in the 'machine'? We only see body stuff. Yes, I can type this email and speak. But this too is all internal. When I die I go back to the place from whence I came, back to the zero before birth. Zero to zero would be my path. I disappear as I appeared. The "I think therefore I am" of Descartes is just begging the question. The "I think" is just that inner I speak of above. Descartes missed it. His "I am" is the "I think". Even he admits the Zero of the outer. The outer doesn't come into it. It's all just the arrangement of the inner atoms/molecules as in my question. A year before my birth I was a zero. A year after my death to come, also a zero. OK, so there was a code with the sperm/egg combining. And that code, that physical entity, controlled the multiplication of molecules unto the full body development. Still, all physical. No? Yes? Thanks for your answer. I do want to write that I've been reading Wittgenstein for years. Most confusing and most edifying. Never stops.

---

You should keep in mind the concept of an emergent. A melody is an emergent out of a set of musical notes; with one arrangement of the notes the melody emerges, with other arrangements it does not. The working order of a machine is emergent out of the parts of the machine - emergent when the machine is properly assembled, not emergent otherwise. And various knots have emergent properties: a reef knot is easily undone, a bowline will not slip, etc. and a knitted sweater (which is a single knot) has the emergent property of keeping you warm. In the same way life is emergent out of chemicals and mind is emergent out of brain. Emergents are just as real as what they emerge from, and usually have new properties not possessed by what they emerge from. Thus life has properties not possessed by chemicals and mind has properties not possessed by brain.

Helier Robinson

back

(6) Noel asked:

Is Philosophy a Science?

---

No. There are two parts to science: theory and experiment. Experiment is empirical, observation dependent. Theory is explanation of experimental results, but must conform to empirical data. Philosophy is like theory, but does not have to conform to empirical data as closely as theory. Theorists are specialists, sometimes described as people who know more and more about less and less until they know everything about nothing; philosophers are generalists, sometime described as people who know less and less about more and more until they know nothing about everything - like Socrates.

Helier Robinson

back

(7) Yolanda asked:

Does philosophy impact Christianity? If so, how?

---

Yes. Two of the greatest Christian theologians were St. Augustine and Thomas Aquinas; the first based his thought on Plato's philosophy and the second on Aristotle's.

Helier Robinson

back

(8) Louise asked:

What is reality?

---

Reality is most often defined as all that exists independently of being perceived - that is, it exists regardless of whether anyone perceives it or not. Sometime the definition is extended to all the exists independently of anyone's mind.

Helier Robinson

back

(9) Richard asked:

There is a vanity today that the major religions have the same God. Is there not a fundamental division, ethically, even within religions in how 'God' is viewed. There is 'God is love' as opposed to the God who issues commands - that is to say, God who is defined in ethical terms, as opposed to ethical mandates that are right because they come from an all powerful God. Or again, ethical values are prior to everything and not subject to the arbitrary power of God.

---

Do not confuse what God is, with what various people believe about God. What God is, is immutable and eternal; what people believe about God is often false.

Helier Robinson

back

(10) Nancy asked:

My daughter's philosophy teacher (high school) asked the class to solve this "riddle" of sorts.

You're in a box. You cannot see or touch the walls, but you know they are there. Your goal is to get out of the box and explain how you did it.

---

If you cannot touch the walls, does that mean that you can walk through them? In that case, you can walk out of the box. If you cannot walk through them then you can bump into them, in which case you can touch them; but you cannot touch them, by hypothesis, therefore it is false that you cannot walk through them. This explains how you did it, once you have walked out.

Helier Robinson

back

(11) Paul asked:

How can we prove others exist?

---

Presumably you mean other minds, not other bodies, of people. If so, then the answer is that we cannot, we can only believe they do. If, on the other hand, you are referring to bodies of other people then we prove that they exist simply by perceiving them.

The essential parts of a philosophical writing are (i) statement of a problem and (ii) discussion of possible solutions of it. For example, your problem might be that of solipsism: does anything exist outside of you present consciousness? You could then discuss various attempts to solve this problem by past philosophers, try to prove solipsism false by showing it to be self-contradictory, or try to prove it false by proving that something exists outside of you consciousness, such as God. For example, Descartes got into solipsism using his method of hyperbolical doubt, which was to discover certainty by doubting everything he could; he finished up being certain only of his own existence, and got beyond it by proving, to his own satisfaction, if not to everyone else's, that God exists.

Helier Robinson

back

(12) Kieran asked:

If the name of a thing is not the thing itself, and the description of a thing is not the thing itself, and the image or mental representation of a thing is not the thing itself, and even the knowledge of a thing is not the thing itself then can the thing be considered/ discussed/ known with any degree of reality? If all things in this way are beyond their name, description, image and knowledge, is this evidence that all things are one? (Owing to the fact that they all essentially lack attributes in a very identical way).

---

I am going to assume that by "known with any degree of reality" you mean "any degree of truth." Then the answer to your question is that our knowledge of things in themselves is speculative. David Hume maintained that " 'Tis vain to speculate" but he was wrong. We have a lot of very good speculation these days, in the form of theoretical science, which tells us about things in themselves by means of quarks, atoms, molecules, etc. But although this is all speculative, it is not evidence that all things are one. They may be one, but theoretical science does not prove it.

Helier Robinson

back

(13) Ty asked:

Which is the philosopher that taught by asking questions?

---

Socrates. If you are lucky you will one day have a philosophy teacher who uses the Socratic method of teaching. She will come into class, but will not lecture - beyond stating a problem. Then she will invite solutions to he problem. Bad ideas will be shot down, good ones will be written up on the board, and the class discussion will be so lively that students will continue the discussion after class, in this coffee shop. That's the sort of stuff you never forget.

Helier Robinson

back

(14) Perdy asked:

Is it possible to talk /communicate/ converse with rocks or trees? What is the answer to this question in classical philosophy?

I'm not interested in the volume on the net of spiritual answers ("call xxx for telepathic inter-species communicators") but whether the question has been considered in terms of language, communication and exchange between two parties.

---

No. Conversation requires at least two minds and a common language. Minds require brains, and rocks and trees do not have brains, so do not have minds, so cannot converse.

Helier Robinson

back

(15) Patricia asked:

Metaphorically speaking, are we any more than just the drop of honey left in the jar, after the contents have been emptied out?

It seems to me that most of us are culls, but we are only necessary so that the wheat can be culled from the chaff.

I don't like the idea of being chaff, but at age 64, I cannot see any difference I have made to contribute to life, in general.

I am married, the mother of two married daughters and six grandchildren. I started the first wildlife rehabilitation group in the county, and it is still going. I ran a welding equipment business in the 60's when a woman was still expected to be in the home. I still don't feel like anything more than the honey left in the jar.

Can you shed any light on the subject?

Thank you.

---

I'm dubious about metaphorically dividing people into wheat and chaff, but if you do then the wheat are the people who leave something they have created behind them when they die, and the chaff are those who don't. For most people their greatest creativity is raising children well. Starting a wildlife rehabilitation group and a welding business also were also creative. So, no, you are not chaff. And the way to think of t he metaphor of honey is that when you are born your jar is full of honey, and as you age the honey gets used up. At 64 you're still a youngster (compared with me) and so are much more that a drop left in the jar. And you may well have some special talent that you don't know about, and which would be worth searching for; if you found it you could be even more creative.email:

Helier Robinson

back

(16) Helier asked:

Qualitative difference entails quantitative difference because whatever A and B may be, if they differ qualitatively then there is some quality Q such that A is Q and B is not-Q; if they are one then one thing is at once Q and not-Q, which is impossible; so A and B must be two. But I am qualitatively different today from what I was yesterday - yesterday I needed a haircut, today I don't - so I-today and I-yesterday are two, cannot be one. What has gone wrong?

---

You are confusing an element of a set with the complete set. If a set A has member a, which is q, and tomorrow a changes to member b which is not-q, then what about the other elements of A? If they haven't changed, then we might say, given a reasonable definition of "same", that A-today is the same as A-tomorrow, right? If you want to be picky, then sure, A-today isn't A-tomorrow... but then of course I-today cannot be I-tomorrow. But that's not the way we mean "same" when we talk about people.

Steven Ravett Brown

back

(17) Patricia asked:

Metaphorically speaking, are we any more than just the drop of honey left in the jar, after the contents have been emptied out?

It seems to me that most of us are culls, but we are only necessary so that the wheat can be culled from the chaff.

I don't like the idea of being chaff, but at age 64, I cannot see any difference I have made to contribute to life, in general.

I am married, the mother of two married daughters and six grandchildren. I started the first wildlife rehabilitation group in the county, and it is still going. I ran a welding equipment business in the 60's when a woman was still expected to be in the home. I still don't feel like anything more than the honey left in the jar.

Can you shed any light on the subject?

---

Offhand it sounds as if you don't need philosophy, you need counseling. What's happening to you is classical depression, as far as I can tell from the little I see here, and I'd strongly advise you to go talk to someone. If you don't want to do that, at least go get regular exercise; it's wonderful for depression. If you want discussions about "the meaning of life", well, we have archives out your ears here. But that doesn't sound like what's really going on, to me.

Steven Ravett Brown

back

(18) Angie asked:

Can God love himself?

---

First letter of John, 4, 8: "He who does not love, does not know God, for God is love"; 4, 12: "(...) his love is among us, and his love comes to live, entirely, among us:

God, in the Christian Bible is compared to love itself. By definition, love cannot love itself, because love is an expression of itself, when it relates to another.

Nuno Hipolito

back

(19) Georgina asked:

I'm studying Mussolini at the moment - can you explain a little about how the philosopher Nietzsche influenced him? Many thanks - Georgina.

---

Benito Mussolini is a fascist.

In reality, true fascism was born and implemented only in Italy, because there it was born from a syndicalistic social background, combined with the Hegelian ideal of what a State should be (following Giovanni Gentile's view). Italian fascism, embodied in Mussolini, was a mix of corporatism, totalitarianism, nationalism, militarism and anti-Communism.

The fascists believed that the State should control all aspects of life, even the spiritual ones.

Nietzsche was not a fascist, neither did he wrote fascist things. He was - that we know for sure - someone who was against all transcendence, theoretical or practical. He wanted to destroy, but only so evolution could take place, towards a "superior culture". Of course, the views of Nietzsche fitted perfectly with the intents of Mussolini, because they could be seen as strictly anti-Marxist, anti-communist.

In his youth Mussolini expressed great admiration for Nietzsche in a work called "La filosofia della forza" ("The philosophy of power"), which title is quite similar to the work of Nietzsche called "The will to power". In this he congratulates the ideas of the philosopher, praising the "superman" as its great creation. Not only to Mussolini, but to all extreme left wing parties and especially syndicalistic revolutionary minds, who hated the bourgeoisie, Nietzsche's ideas were very appealing.

Of course Nietzsche did not intend, so far as we know, to have this effect in the future, at least not the effect it turned out to be - as a strong theoretical influence mainly in Nazism and Italian Fascism. He was long dead when Hitler came to power. The problem is that, the beauty and vitality of his ideas, very often poetic, were not seen as a system of ideas, and so were able to be interpreted in various manners, including the Fascist/Nazism one; of a "superman" that was really an embodiment of a "super-race". Although in Italian Fascism this idea of the super-race was not implemented in such a way as it was in the German Nazism. In fact, to Mussolini, the "will to power" was much more important, as a basis for his social revolution.

Nietzsche was, in short, a very strong spiritual influence on the young Mussolini, and the perfect philosopher for him to base his totalitarian view of the State.

In fact, for everyone who thinks philosophy has no real life applications, that think philosophy is just a waste of time, this is a serious matter to reflect upon.

If you can read Spanish there is a very interesting link you can visit:

http://personales.ciudad.com.ar/f_nietzsche/instantes/nietzsche_mussolini.htm

If not, just read any article on Italian Fascism, to learn more.

Nuno Hipolito

back

(20) Patricia asked:

Metaphorically speaking, are we any more than just the drop of honey left in the jar, after the contents have been emptied out?

It seems to me that most of us are culls, but we are only necessary so that the wheat can be culled from the chaff.

I don't like the idea of being chaff, but at age 64, I cannot see any difference I have made to contribute to life, in general.

I am married, the mother of two married daughters and six grandchildren. I started the first wildlife rehabilitation group in the county, and it is still going. I ran a welding equipment business in the 60's when a woman was still expected to be in the home. I still don't feel like anything more than the honey left in the jar.

Can you shed any light on the subject?

Thank you.

---

First of all, it seems that you are in a mid life crisis, albeit in your mid sixties. Maybe you're a late bloomer, just like me.

You say you didn't make an impact in life in general, but then you say that you had two daughters, six grandchildren, ran a business, etc...

Do you really feel unhappy with your legacy or do you feel alone, because your daughters are gone (in a symbolic sense, of course), and you based your life too much around doing things for others?

From your words, I can sense this is what really bothers you now.

But don't worry. Just think about all the things you can do, things you are interested in, and things you didn't have the time to do before. Do you have interests? Do you feel you could make a difference in your community with your free time? Could you also focus on helping your daughters, raising your grandkids? Seems like you have a lot of options.

I have said this here before: you can't measure a life by what you accomplish in it. If it was so, only Alexander and one or two Roman Emperors would have lived a full life.

If you feel alone, don't let it depress you to a degree that you think you didn't make a difference. I'm sure your daughters disagree. And you are not dead yet go out there and start doing the things you feel you should be doing, to feel better about yourself.

Nuno Hipolito

back

(21) Gerald asked:

What are the differences and similarities between society and culture? Are they interchangeable words? Are they concepts that have no boundaries but are enmeshed?

---

I think they became enmeshed, as you say, over the time.

You can't have society without culture. But you can have culture without society, can't you? Just imagine Robinson Crusoe, stranded in his deserted island. If he makes something with his hands, if he writes a book or a symphony isn't that culture, and society is nowhere to be found.

Nuno Hipolito

back

(22) Kyle asked:

What are some of the reasons I should not buy things from companies with ethics I don't agree with?

---

The reasons could be moral, or ethical ones. If you lead your life according to some moral principles, they might stop you from buying certain things.

You could be an animal lover, and therefore disagree from animal product testing. If you find a brand that tests its products on animals, you should feel strongly against buying their things.

Nuno Hipolito

back

(23) Khizar asked:

Which fifteen philosophical books would you consider to be the most important/influential ones ever written. Thankyou

---

This is like asking someone about their favourite movies...

This is my personal list, and is, of course, a very subjective one.

1. The Bible (various authors)
2. Plato's "The Apology of Socrates"
3. Plato's "The Republic"
4. Aristotle's "Physics"
5. Saint Augustine's "The City of God"
6. Machiavelli's "The Prince"
7. Descartes's "Discourse on Method"
8. Julien De La Mettrie's "The Man Machine"
9. Kant's "Critique of Pure Reason"
10. Hegel's "Phenomenology of Spirit"
11. Nietzsche's "The Anti-Christ"
12. Nietzsche's "Thus Spoke Zarathustra"
13. Fernando Pessoa's "The Book of Disquiet"
14. Ludwig Wittgenstein's "Tractatus Logico-Philosophicus"
15. Jean-Paul Sartre's "Being and Nothingness"

Nuno Hipolito

back

(24) Stacy asked:

Hi I have invisible arms under my belly that no-one can feel am I an angel and did god give me this.

---

This sounded so cute, it just made me smile.

I just hope this wasn't posted by a 40 year old man.

If you think you have invisible arms under your belly, well, I don't think angels had arms, they had wings. You should have a closer look at them, and see if they are indeed arms, or wings.

If they are wings, there's a pretty good chance you're an angel.

If they are arms, you are probably just a mutant, which is pretty cool in itself.

Nuno Hipolito

back

(25) Jed asked:

Enlightenment is defined by Therevada Buddhism as an acknowledgement of the cessation one's material, fine-material and non-material existence. What philosophical question would be meaningful to this kind of person?

---

Well, an ontological one, for sure. But also a gnosiological one.

Buddhists defend that knowledge comes from the obliteration of the self. By destroying our material self, our spiritual self would arise, eventually. It's not really the need to deny the material self, only the necessity of denying that self all the importance and attention it gets.

Let's face it, we do grant too much importance to material things.

A Buddhist poses tough questions like: are material things important to me? Can I find happiness without a fully material existence? Can I be all I can be, if I continue to pursue happiness only in a material existence? These are ontological (being) and gnosiological (knowing) questions".

Nuno Hipolito

back

(26) Helier asked:

Telescopes and microscopes do not enlarge reality, they only enlarge images of reality. Everything seen through a lens is an image of reality, not reality. But our eyes have lenses, so everything we see is only an image of reality. Can this be true?

---

It is certainly true that we build an image of reality, of course. If we had infra red capability, our reality would be much different than what it is. If we only sensed heat, our lives would probably revolve more around heat than colour. We would wear clothes that reflected that, and probably we would look hideous in them.

But your question seems to ask if there is a "true reality" out there. I don't think so, because reality depends on perception, and perception varies, accordingly to whoever perceives reality. So there are different realities, because there are different perceptions of reality.

Of course you could argue that reality exists without perception of it, but just remember that "reality" is a human concept.

Nuno Hipolito

back

(27) Helier asked:

Is it true that in science 'theoretical' means 'non-empirical'? If so, are theoretical entities radically imperceptible? That is, although we can perceive the effects of theoretical entities, we can never perceive the entities themselves. For example, theoretical temperature is average kinetic energy of molecules, which we cannot perceive, but we can perceive its effects as thermometer readings and sensations of hot and cold; or mass is imperceptible but we can perceive its effects as forces of weight and inertia.

---

In science "theoretical" does not always mean "non-empirical".

Even in such fields like "theoretical physics", often hypothesis are proven later on, and seen as scientific demonstrations. Black holes where theoretical until one was seen.

Nuno Hipolito

back

(28) Shelley asked:

I am actually searching for the two famous people that had a conversation which included this paraphrased question: "What does it profit a man if he gains the whole world but loses his soul?" Thanks, I hope you can help me

---

That quote is actually from the Bible, Mathew, 16:26.

Eugene O'Neill, a famous American poet and playwright, thought it was the secret to human happiness.

Nuno Hipolito

back

(29) Leah asked:

Are we free if we don't know it?

---

Yes, its called childhood.

Nuno Hipolito

back

(30) JF asked:

What does it mean to love?

---

No one really knows what love is, and, for some, therein lays love's greatest value and mystery. In general terms, being in love, means you are devoted to someone else, and you find in that person comfort and acceptance, making you a less selfish person, for you no longer search your own well being, but also the well being of your loved one, at times at the expense of your own.

Nuno Hipolito

back

(31) Zyryl asked:

Is there such thing as nothing?

---

Yes. Nothing is a human idea or concept. It exists as a concept, even if you defend that "nothing" cannot exist as "nothing".

There were two primordial theses on "nothing".

Parmenides said that "the nothing does not exist". Plato, on the other hand, said that "there is being in non-being". This is because Plato saw that, for ideas to come and go, they had to "evolve" from and into constant states of change, of motion, between being and not-being.

For Parmenides we could not know the "nothing".

Plato said that the "nothing" exists, but as the negative of the "being".

Nuno Hipolito

back

(32) Georgina asked:

I'm studying Mussolini at the moment, can you explain a little about how the philosopher Nietzsche influenced him. Thanks, Georgina.

---

I know Mussolini read Nietzsche and apparently wanted to write a book on him. Mussolini may have been personally influenced by Nietzsche's writings but I don't know specifically how. Nowhere, in Italian Fascist Ideology as I am aware is there a definite aspect attributable to Nietzsche. Nietzsche opposed the all powerful state [Zarathustra: Of the New Idol], herd morality, [Beyond Good and Evil: 199,202,203.], Militarism and Nationalism [Beyond Good and Evil: Peoples and Fatherlands] all of which are central to the Fascist Ideology. Also, its insistence on socio-political holistic homogeneity is characteristic of modernity and Nietzsche was critical of this. He preferred a plurality of creative free spirits who would be the commanders and lawgivers of their perspectives, 'truths'. So pluralist dynamic of values would exist. This is the antithesis of the expected uniformity and blind obedience of the people to the corporate Fascist state.

If Nietzsche appears to sanctify aggression and expansionism [BGE 257, 258] citing it as the will to power and drives, this has to be qualified with his emphasis on spiritualization of drives. The will to power, drives have been sublimated and spiritualised. So much so that your desire to satisfy a curiosity and my desire to write and to hopefully answer your question are both spiritualised manifestations of the will to power. The spiritualization and sublimation of the will to power and drives is often overlooked by readers who then cite Nietzsche as the philosopher of war, struggle, domination, the justification for Hitler and so on.

Nietzsche also appears to praise Ceasare Borgia, Napoleon which can give the impression he favoured a 'strong man' or leader. Again, Nietzsche cites these individuals to highlight the vitality; the daringness he believes is lacking from contemporary society this being consequential upon the heritage of Western Christianity.

See Walter Kaufman. Ch 10 Nietzsche: Philosopher, Psychologist, Anti-Christ. Princeton University Press 1974.

Martin Jenkins

back

(33) Malvik asked:

What contribution did Nietzsche make to the Philosophical understanding of Truth?

---

Metaphysical Truth

The weary herd found life hard and solace was provided by a priestly caste that cited the meaning, the truth of life as residing elsewhere. [Genealogy of Morals 1]. These 'Afterworldsmen' [Zarathustra. Of the Afterworldsmen] postulated the origin of life, the world, of beings; the Truth of all that is, as lying beyond in an a-temporal, infinite realm. Downgrading 'this-world' as transient therefore unreliable as a source of knowledge, the Truth was transposed to the up graded 'other-world'. [Twilight of the Idols. Reason In Philosophy]. Upon the adoption of this slave morality and epistemology, Western Philosophers sought certainty in 'The Forms', in 'Essence', in the theistic most real being that is God. Being and Truth emanated from these sources to the lesser world. Belonging to the body, which is part of the world, the evidence of the senses are condemned as unreliable. Knowledge and Truth can only be obtained by the soul practicing Reason. Only Reason can provide Rational Truth by the application of clear, dispassionate analyses. Hence logic, mathematics, theology and metaphysics are the sources of Truth. Yet metaphysical methodology did not deduce a single, Absolute Truth. Take a look at the history of Western Philosophy with the different applications of the 'correct method'. So when Nietzsche is proclaiming the death of God, he is rejecting the metaphysical approach and paradigm.

Will to Truth is Will to Power

Will to Power is the dynamic of Life. It aims at enhancement and expansion. Knowledge is judged not on its inherent coherence [a-priori metaphysics] but to the extent that it preserves and enhances life [B.G.E. 4]. Hence Western Philosophy, Metaphysics and its 'Truth' can find its genesis in the preserving instincts of decadent beings. The paradigm of Western thinking followed upon this origin and has correspondingly limited Thought and pushed it in the direction of metaphysics. Nietzsche believed the moment to expose and transcend metaphysics had arrived. The 'truths' of metaphysics would fetter human development and flourishing. Hence monotheism with its uniformity of all before the One Truth would succumb to Perspectivism expressive of letting a thousand will to powers to flourish. Perspectivism is not a matter of maintaining the fable of Truth being objective and 'out there' but is constructed by human beings. The development of 'truth' is now a matter of epistemological conversations grounded in human beings, interests, needs within the hegemony of the dominant discourse which at the moment at least, provides 'the goods' Think of the competing claims of capitalism, anti-capitalism, ecology and profit, of the best way to run a community, a school, an office, between friends, between enemies. This is controversial to say the least!

See: Maudmarie Clark. Nietzsche on Truth and Philosophy. Cambridge University Press 1990.

Martin Jenkins

back

(34) Zyryl asked:

Is there such a thing as Nothing?

---

If Nothing is the absence of a 'thing' then this absence is understood, at least linguistically. But if being is some thing then no thing is the opposite of this: i.e., non-being or nothing. Then nothing could not exist, as it is not in Being. Likewise, although I understand the term 'unicorn' it does not follow that it exists - not being a subject of possible experience. Arguably, nothingness can be experienced and the very fact that its existence has been posed in a question leads beyond analytics to Phenomenology.

Phenomenologically, Nothing exists as it is capable of being experienced. In What Is Metaphysics? Martin Heidegger writes that nothingness is experienced upon anxiety. In this mood the nihilation of beings means they seem to fall away, melt from existence and nothingness shows itself. So nothingness is something in order for this to happen. In Being and Nothingness, Jean-Paul Sartre proposes that Nothingness is constitutive of Being. Being is surrounded by Nothingness. Failed expectation allows Nothingness to arise when, for example, a friend fails to appear at an arranged meeting. His absence allows Nothingness to arise. G.W.F. Hegel in his Logic cites Non-Being of Nothingness as the opposite of Being. Both stand opposed to each other. As the term non-being is subject to understanding, it cannot be nothing. Nothing possesses a being although very empty. The movement between these two notions is a Becoming that links them. So Nothing is some thing.

Martin Jenkins

back

(35) Ian asked:

Is it really possible to be a philosopher and a christian at the same time? How can a philosopher be asking any Meaningful questions if at the heart of him/ herself he/ she believes that, apparently 'out of the blue' an all knowing etc Superbeing just thought it all up as a self-aggrandising project?

---

Before a philosopher asks any reasonable question, much less a meaningful one, he/she has to have a reasonable degree of knowledge of the topic(s) that he/she seeks to explore further. No philosopher could possibly entertain an understanding of Christianity anything like the one you outline.

For evidence that philosophers can seriously consider questions relating to Christianity and pose meaningful questions about its beliefs you could look at my paper "An Emergent Christology" on my Web Page at http://www.philosophy.27south.com

To limit my reply to your first sentence; The answer is simply yes, it is possible. It happens quite frequently.

Tony Kelly

back

(36) Stephanie asked:

Is life really over when it ends? Why do we exist; We get born, we cry and laugh and then we die... Why?

---

When I was involved in criminal investigations I suspended judgement until had considered all the available evidence. Investigators, including scientists and philosophers, function on the assumption that everything that happens does so for a reason.

People have been born and dying for 160,000 years since Homo sapiens evolved. Originally the species was not much different from other animal species except, like other Hominids, sapiens began to form cultures.

Cultures are forms of self-creation. People form cultures and cultures help form the people of the culture. The present human situation is vastly different from the situation of the original Homo sapiens, and we are vastly different from them. In the last 3,000 years we have begun to learn to think critically and to become moral. (see Bruno Snell's "The Invention of Mind".)

Living, thinking new things, changing the world, passing things on to a new generation before passing on yourself, seems to have achieved something. Whether we achieve only the right things is the question we all should ask.

Tony Kelly

back

(37) Tony asked:

Why are we here, is there a reason or purpose or is it just chance? Chance seems to me to be the best explanation, but then what?

---

Chance is no explanation. Ontologically there is no such thing as Chance, any more than there is such a thing as a Unicorn. In a sense we all know what Chance is, just as we know what a Unicorn is, but neither exists. When we toss a coin it falls heads or tails by "chance". But if we make a device to toss a coin and it imparts the same force every time, the coin will fall the same every time. Chance equals "I don't know".

There is a reason for us being here because there is a reason for everything. Consider the evidence and try to work it out. See also Answers 19, No. 84.

Tony Kelly

back

(38) Evelyn asked:

What is the meaning of life?

---

We are fortunate to live at a time and in a society that allows more freedom of thought and action than ever before. What our own life may mean is very much our own responsibility. The most important thing we will ever make is our self. That you ask this question indicates you are on the right track.

Tony Kelly

back

(39) Jayson asked:

Is it true that the 'existence of the external world is dependent on individual's perception'?

---

It is certainly true that our perceptions are subjective, we could say that this is evident in our every day lives when we find people engaged in arguments brought about by different views of the same thing. Objects remain the same, but views about them differ because they are perceived in different ways. If there were no subjective views and everyone was geared by nature to fixed objective views of the world there would never be any arguments, and wars would be unknown.

A simple way of considering the subjective element is by way of psychology. To use a basic example, a person born and brought up in a poor and deprived environment will see the world very differently to a person born and brought up in affluent privileged conditions. To one the world is a bleak, desperate place where one is involved in a daily struggle to survive. To the other the world is a wonderful place, providing enjoyment, opportunities and privileged status.

One important philosophical notion is that of the 18th Century philosopher, Bishop Berkeley. This is the view that to exist a thing must be perceived. According to Berkeley the only things we can consider to be real are ideas within the mind. He considers the notion of 'matter' to be nothing more than an invention to come to terms with our daily sensations. These sensations occurring within the mind are the only 'real' things available to us, all things/ideas brought to mind are thus perceived, they are things of immediate acquaintance dependent on perception. When we divert our perception from, say a tree, the tree does not go out of existence because God continues to perceive it. This 'idealist' view has lost a great deal of support, however it is still a notion which some find difficult to refute. However, perception can be understood to be a basic ingredient within all schools of thought in philosophy. Whether or not we can say that the 'existence' of the external world (if indeed there is such a thing) 'depends' on individual perception remains a debatable philosophical problem. The great 18th. Century German philosopher. Kant was convinced that the 'reality' which provides the empirical data/sensations of our everyday lives can never be actually known to us, we exist within a world of perceived sensations divorced from the 'real' world which provides the sensations.

John Brandon

back

(40) Stephen asked:

What makes one explanation more plausible than alternative ones? And if one is more plausible, how does this make it more likely to be true? I can see sort of how it works but I'm unable to put it into words. For example, if I find what appear to be cycle tracks in the dirt, what seems to me to be the most plausible causal explanation would be that a cyclist has rode through. But why is this explanation more plausible than, for example, two people rolling bike wheels? Or even that the dirt just happened to form that shape by itself? Is there a way of evaluating different explanations? And, if there is, does this apply to other kinds of explanations (e.g. purposive.) I've read that the best explanations are ones which increase our understanding, but how does this make them more likely to be true?

---

This is a very good and extremely difficult question to answer, in general. One can talk about probability and likelihoods until the cows come home, but in the end induction is just uncertain, and there's no way around it. But if you do take induction as having value, then you've got a starting point... you can indeed say that there are "more plausible" or even "more probable" explanations for things, depending on the history of observations of similar occurrences. But this field both has an enormous literature, and that literature, by and large, is very difficult. Statistics, probability, philosophy of science... here's one of my favorite books in this field, but it only touches some of the most contentious points, and it's not a particularly easy read (but compared to some, it's a walk):

Kitcher, P. The Advancement of Science; Science without Legend, Objectivity without Illusions. New York, NY: Oxford University Press, 1993.

If you're still interested after reading this, go look at some of the readings he cites.

Steven Ravett Brown

back

(41) Matt asked:

Can Omniscience and "Free-Will co-exist? Don't worry this isn't a homework assignment; I was just recently looking up things about it because of a discussion I had in class one day with my teacher. We thought that we had put the issue to rest when we established that they could not because if God is omniscience then he knows everything and would already now the outcome of the decision you are going to make, but I have since been reading other answers to the question which suggest that we have free-will because God does not influence the decision he merely knows the outcome of that decision. That to me though doesn't really make any sense, so I was hoping you might be able to shed some light on the matter.

---

Well, look, first, I'm not at all a theist, so the whole "god" thing is a bit bizarre to me... but I've never understood how any of the infinite attributions of any particular god (yours being, I assume, Jahweh) can be made to work. Can "god" lift a rock too heavy to lift? Does god know everything, even the things that it can't know (I mean, Armageddon? Think about it.). The only group who took the above seriously and logically worked it out, as far as I know, were the Puritans, who maintained that their god did indeed know everything, including the kind of life you were going to lead, and so at birth your heavenly or hellish destination was already predetermined, and the rest was just regretting what was pretty much inevitable. No wonder they wore black and looked so depressed, right? All the other theists who have given any thought to this (e.g., Aquinas, etc.) have concluded that their particular god basically, as far as I can understand it, knows everything as it runs, but has given us the ability to get out from under its infinite foreknowledge because of its infinite wisdom, mercy... something like that. As you can see, I'm as puzzled as you are, really... but the same kind of thing holds for any infinitely large, strong, powerful, intelligent, etc., etc., characteristics: the paradoxes inevitably arise. But is this surprising? Go here for a more complete list:

http://www.infidels.org/library/modern/jim_meritt/bible-contradictions.html

Steven Ravett Brown

back

(42) Chris asked:

If artificial intelligence challenged God to a chess game, and neither one could cheat in any way, would it always end up a stalemate? Or could God think of a way to defeat it?

---

Haha... I'm afraid not. If both stuck to the rules, and both played ideal games, the one making the first move would win or draw. It's a proven theorem in Game Theory that the player who moves first in a two-player, zero-sum game (win=1; lose=-1; draw=0) can't lose.

Steven Ravett Brown

back

(43) Stephanie asked:

Is life really over when it ends? Why do we exist: We get born, we cry and laugh and then we die: Why?

and Tony asked:

I have no belief in a god or gods or an afterlife.I often struggle with the question of being. Why are we here, is there a reason or a purpose or is it just chance? Chance seems to me the best explanation, but then what?

---

There is a relationship between these two questions, and that is a hint of pessimism. Neither Stephanie nor Tony can see any point in their existence, this I find rather sad.

Let us separate out the questions posed.

(1) What do we mean by being?
(2) Why do we exist?
(3) Do our lives have a purpose?
(4) Is life an accidental event?
(5) What do we mean by death?
(6) Is there an afterlife?

Perusal of this list reveals that we are dealing with questions that have taxed the minds of philosophers down the ages. Unfortunately I am unable to place a list of answers alongside the questions, no such list exists. There are lots of hypotheses, enough to fill several volumes in their extended forms, they range from what appears to be solid science to a fascinating trip through fairyland. Philosophy in general might claim that it has done the major part of the job by identifying the problems. Identifying problems is one of the major functions of philosophy. Some philosophers, for example A J Ayer, might question the questions and ask if we are indeed asking questions which we are capable of answering. A philosopher like Kant would say that there are questions in this list that man will never be capable of answering. So why have I even bothered to concern myself with these questions? I think because of the pessimistic approach, pessimism always stirs me to respond; it must be something in my psychological make-up. In this case I am disposed to ask of Stephanie and Tony why, when they have a choice, they choose a pessimistic view rather than an optimistic one? Why not ask questions about enjoying life? Why not be excited by the fascination of life? Why don't we all wake up to the fact that life is what we make of it? Consider life to be pointless, make no mistake it will be. Consider life to be exciting and worthwhile, make no mistake it will be. We can live life in a shoddy, superficial, way, or we can seek for depth and purpose in life. If our lives today seem pointless and futile, hard lines, we have chosen the wrong path, nobody chose it for us, though this sometimes seems to be the case. Self reliance is something to be sought, it is an essential foundation for life. Difficult to achieve for some, but well worth pursuing.

Taking the questions in turn, most thinkers would claim that to be is to exist. A human being is then something or someone that exists. We could extend this to say that a human being is a being that exists within a world of existing things. Now, as for the questions: Why do we exist and is it for a purpose? There seems to be no one out there that can tell us, though this is something which religious believers would dispute.However, we do seem to have been gifted with a mind, making available to us an inward search. In our inward search we may find personal ambition, a desire to do something, to achieve something. The question: Is there a purpose in life? can only apply to the individual; detecting a purpose for all humanity is very much beyond us at this time. Is life an accidental event? The optimistic view is no. Order in the world suggests a plan. However,for most scientists the claim is for a rather dubious series of accidental events.

What do we mean by death? It could mean that we have come to the end of this phase of our existence.It could mean a simple transition to another sort of existence: and, yes, it could mean the end for the individual. Who knows? There is, however, one thing to bear in mind: a person who fears death dies many times, a person who has no fear of death dies only once.Is there an afterlife? Most believe that no one has ever come back to tell us, but there are those who are convinced that this has happened. Again, we are dealing with individual experiences, individual claims. How are we to deny that someone has had a spiritual or religious experience? How do we know that the existence we have been discussing does not include other beings different to the ones we encounter on a regular basis? One great lesson that most philosophers learn at an early stage is not to take too much for granted. Proof can be so elusive, but we have no right to claim that it can never be available.

John Brandon

back

(44) Jadey asked:

Do animals believe anything? He seems to know when he's going to be fed and he gets exited when my dads car pulls up, almost as if he knows who it is from just the sound. He can even work out little problems like how to get into our neighbor's garden. But its difficult to imagine him believing and knowing things without having a language. We always say things like "I bet he thinks he's going for a walk," but does he really THINK?

---

Your dog seems to be excited about things he "knows" will happen, because those things always happen, everyday, it's kind of a routine for him. So his reactions are not a real sign of "knowing", rather an intuitive response. His brain expects them to happen everyday. If one day your dad changes the time he comes home, your dog probably will still wait for him at that time.

Animals, such as dogs, do think. The problem is, they tend to think at a very basic level. Your dog is probably more dumb (sorry for the use of the word) than a 6 month year old child. And that's about it for him; he won't get any smarter than that, because of his brain".

Nuno Hipolito

back

(45) Bob asked:

What is a philosopher?

---

A philosopher is someone who loves wisdom. A love is a very sincere thing, so if this philosopher uses his knowledge to get all arrogant, he probably his betraying his love for his arrogance.

back

(46) Catherine asked:

What are some of the philosophical issues surrounding Nazism?

---

I see two main ones:1) the segregation of the Jews following the concept of a super-race; 2) the persecution of the Jews by establishing anti-Semitic laws.

Issue 1) relates mainly to the views of Nietzsche, but comes from an older, much significant, Nordic tradition of the superiority of the Nordic people, in comparison with southern people. This probably arises from early on, because of the invasion of the northern territories by tribes from China and Mongolia.

Issue 2) brings the very interesting question of immoral laws, or illegal laws. Laws were made (and approved in parliament) to accommodate the persecution of the Jews and the cleansing of the race in Germany. This had a very shocking effect in society: disabled kids were killed, mental patients were sterilized, Jews were sent to ghettos, etc.

Nuno Hipolito

back

(48) Bo Peng asked:

What are the dangers of basing business decisions solely on financial statements?

---

The danger is quite obvious: a business has social responsibilities in its community. And the bigger the business, the larger the responsibilities.

Businesses employ people, people that constitute the community in which the business operates and makes money. It's important that the business decisions are made taking that into consideration, for PR reasons, but also for moral reasons.

A business should evolve in balance with its surroundings, and sharing its growth with the ones that contribute more to that growth. Employees are a important part of a business, and they should not be ignored, when it comes to sharing profit, for example.

Nuno Hipolito

back

(49) Richard asked:

Democracy derives from the Greek meaning government by the people, and even in Athens this was very selective. Fifty years ago JF Strong wrote that it is a term applied to so many states, "it no longer helps us to a division of them". He was no doubt thinking of the myriad constitutions. However, might not a philosopher consider definitions in terms of basic ethical values? May not democracy be considered consonant with freedom, in opposition to tyranny? In that case, might it not be divided into two essential forms, that is anarchism and altruistic democracy? The essential difference being perhaps that anarchism contains no social responsibility, above the level of non-interference, while altruistic democracy would have such responsibility to society and to people in general?

---

Democracy actually means Power (cracy) of the People (Demo).

You are right that, in the present times, we can't tell very well what sets democracies apart anymore. At least in Europe, we now have a long tradition of democracy, translated in social liberties, citizenship, and freely elected governments.

I think a democracy exists when there are free elections. That's the basis for everything else. But there can be free elections and not a democracy. Many states have free elections, but those are conditioned, or only some people can be elected, making them fake elections.

I don't thin you can say that a democracy is only an expression of freedom. For instance, in the USA , you have the self proclaimed best democracy of the world, but you have a correspondent increased pressure on individual rights and liberties.

Essentially modern democracies are the same. They only differ on the kind of social state they implement.

Nuno Hipolito

back

(50) Richard asked:

I want to ask you what is the significance of a coincidence? The other day at dinner my wife was trying to remember the answer to a children's riddle, "what noise annoys an oyster?" Two hours later (and I had never heard this riddle before) I found the answer in an suspense thriller I'm reading set in Moscow ("a noisy noise annoys an oyster"). A scientist might say I was sensitized to finds this coincidence and unaware of all the myriad times a coincidence does not appear. I'm not satisfied with that explanation. The odds of finding the answer above must be extraordinarily high. What do you make of high odds coincidences?

---

High odds coincidences only occur very far apart from each other, ever wondered about that? That's because they are very rare.

And you remember when something very rare occurs. As you remember this episode, with the riddle.

But some people don't believe in coincidences. If you are one of those, you would tend to say that things always happen for a reason. Of course this is very difficult, if not impossible, to prove scientifically. I believe studies proved that coincidences are statistical events, which are explained by modern science.

Just remember that science is the new religion. It's never a good thing to be an extremist.

Nuno Hipolito

back

(51) Fiza asked:

What does it mean to be an educated person?

---

It depends immensely on the people around you.

If you live in a tribe, which has stone age instruments and believes fire is a divine force that comes from the sky, being able to read must seem like a very strange and distinguished feature to possess.

On the other hand, if you live around people with PhD's, being able to read could mean you are extremely under-educated, almost ignorant.

Of course, there is the other kind of education: being polite and well mannered. This is also a kind of social adaptation knowledge, which varies, depending on your social surroundings.

So you are educated in accordance to your surroundings, basically.

Nuno Hipolito

back

(52) Diana asked:

I have essay due next week and I still haven't decide what topic I'm going to write on my essay. I started the topic "abortion" I think that's like very controversial. Right now I'm thinking to write about "gay marriage". My question is which topic do you think is best? Or do you have any suggestions. May be you can give me some reference.

---

I find it funny that you think abortion is controversial, so you go to... gay marriage!

So... gay marriage is not controversial?

Anyway, why do you want a controversial theme? You should probably find a theme that you are interested about, one that makes sense for you to write about. Ask yourself what are your interests, your hobbies, things you see happening in your community, or your country, things that affect you, and write about one of them. If you write about something you are passionate about, you'll see that the writing will flow much better.

Nuno Hipolito

back

(53) Myles asked:

In the ever-present wake of such corrupt exploitations and tragic misuses of Western philosophy in general: From Rousseau's philosophy contributing to the French Revolution, from Nietzsche's philosophy contributing to Hitler and the Fascist mentality, from Marx's philosophy contributing to Stalin and crooked Communism abroad; from even Leo Strauss' philosophy contributing to the imperialistic Bush Administration. Is philosophy as beneficial to history as most if not all academics would agree upon, or has it been more destructive too mankind then we actually realize, especially when these individually created philosophies are almost always haphazardly and systematically applied to the more collective, political sphere?

---

"You forgot some little details, like Socrates's philosophy contributing to free will and individual freedom, Nietzsche's philosophy contributing to undermine the church's power in Europe, Sartre's philosophy contributing to individual freedom of choice... Sure some of them had pretty radical ideas, but that's the reason they were considered original, and bold.

I, for one, wouldn't confuse the ideas with the ones that put them into practical use. The French Revolution was very messy and even corrupt, Hitler was a insecure egomaniacal failed artist, Mussolini was an idiot, Stalin got drunk on power.

I think philosophy is, at times, good and bad, as all knowledge and research into human things. Physics is good when we find a new planet, and it is bad when we develop a new neutron bomb... all things have (at least) two sides.

The problem with philosophy is that often new thoughts come in waves of revolutions. And revolutions are often messy affairs, because there is destruction involved. But some good things often come out of revolutions. So I wouldn't be so harsh in judging philosophy and philosophers.

Nuno Hipolito

back

(54) Nicole asked:

These are questions asked by my 10th grade English classes. I know there are a lot of questions listed here, but they would be very excited if you guys could respond to a few, maybe over time. Thank you!:

1. Is there a parallel universe?

2. Has human nature changed over time?

3. Why do people have emotions and animals don't?

4. To what extent can morality prevent the destruction of the world at the hands of mankind?

5. What is the meaning of human existence compared to animal existence?

6. If we lived in a perfect world, would there be people tempted to mess it up?

7. Are people naturally good or evil?

8. How will the world end?

9. Does a person's environment decide one's morality or does their personality?

10. Is it really better to have loved and lost than never to have loved at all?

11. If we were always happy, would we really be happy?

12. Is the existence of one state of mind or emotion determined by the existence of it's opposite?

Thank you again. The students are really excited to hear how philosophers would respond to these.

---

1) We don't know. Also, what does "parallel" mean in this question?

2) Yes. Humans have evolved to become more domestic animals, because of higher population density, wars, and so forth. We are more tame than our remote ancestors.

3) Animals do have emotions. There's lots of literature on this. But it's another question as to how they're aware of their emotions. See 5.

4) Haha, good question.

5) Humans are more self-aware than other animals, as far as we know, and can reason reflexively, i.e., can use recursion in rationality. This makes us able to ask questions about "meaning", to be more self-aware of thoughts and feelings, to use symbols, and so forth.

6) What's a "perfect world"? If it's one where people aren't tempted to mess it up, then the answer is no. Otherwise, probably yes.

7) Neither. There's too much interaction from culture to make genetics dominant here.

8) The sun will slowly expand and the solar system will be absorbed into it. But don't hold your breath. Also, if we aren't able to escape by the time it happens (eons from now), we're not worth getting upset about.

9) Yes. Both. See 7. 10) You can only answer that for yourself.

11) Yes. We have both pleasure and pain centers in the brain; so pleasure is not completely relative. Neither is pain.

12) No. See 11. The same holds for most other basic emotions. More complex ones, like irony, are open to debate on this... but probably also innate, because there are stages when they develop which are fairly standard in one's growth.

Steven Ravett Brown

back

(55) Robert asked:

1. Is it ethical to search for illegal copies of intellectual property over the internet on stranger's computers?

2. If it is ethical to search for illegal copies of intellectual property, then is it ethical to erase that data from said computer?

3. Is it ethical to leave computer programs on strangers computers that detect and report back if illegal intellectual property is found?

4. Is it ethical to design into the computer processor itself, a registry that when not set by a proper key prevents the execution of illegal intellectual property?

If the answers to 3. and 4. are different, then why are they different?

---

1) I would say not without a warrant, since you're effectively breaking and entering.

2) No, because now you're not just breaking and entering, you're stealing/ destroying property. I don't see your destruction as justified by another's stealing. This is the kind of thing we have police, courts, and law for.

3) Not without their permission, or the permission of the computers' owner(s).

4) A more interesting question. I'd say yes on this one, with the difference between 3 and 4 being that people buying the computer should be made aware of this before they buy. They then have a choice, which (3) does not give them. But why not do this the way it is currently being done, i.e., with passwords, serial numbers, etc.? Yes, they can be circumvented, but don't you think that your solution also would be? I see a huge black market of computer chips without the registry you're talking about, and/ or PROM burning which circumvents it.

Steven Ravett Brown

back

(56) Mark asked:

How illusion and errors are formed in the Plato's Allegory of the Cave?

---

As Plato presents the Allegory of the Cave in his Politeia (Book VII, 514-518), he likens the sense-impressions to the shadows and the reflections of the things themselves, which is what the people of the cave can see. On the other hand the man who has escaped from the cave (visible realm) to the intelligible realm, he can see the light of the sun, namely he can achieve the knowledge of the Good itself, since he has knowledge of the intelligible Forms (Ideas). This follows from his previous use of the analogue of the sun (Book VI, 506-508), where the sun corresponds to the Good, the light to the truth, and the visible things to the intelligible Forms (noemata), since they are modeled on the Forms.

Then the man who has knowledge (episteme) of the Forms, he possesses the understanding power of the intellect (nous) the analogue of the eye-sight which he calls noesis (understanding) in the analogue of the line that follows in the 509-511. On the other hand, the man of the cave, since he perceives only the sensible things (shadows), without having understanding of their underlying intelligible Forms, he can only have a belief (pistis) or an opinion (doxa) about them. Therefore he has an illusion of knowledge.

However, Plato in his later Dialogues Theaetetus (208-210) and specially in the Sophist (263-268) he makes more clear the concept of knowledge with regard to the correct and the false judgements .

Nikolaos Bakalis

back

(57) Jessica asked:

Why do we park on a driveway and drive on a parkway? also that little indestructible black box on the planes, why don't they make the planes out of that stuff?

You mean you never drive on a driveway? Or park on a parkway? Oh dear. As for the box, if you made planes out of it they'd never crash because they'd weigh too much to get off the ground.

Steven Ravett Brown

back

(58) Heather asked:

In the dialogues Meno and the Phaedo, Plato argues for the doctrine of Forms, the immortality of the soul and the acquisition of Knowledge in this life as a process of recollection. What is his argument?

---

Plato proves the immortality of the soul, so to say the persistence of the soul after the death of the body, in his dialogue Phaedo, by the use of four arguments: The cycle of opposites, the theory of recollection, the resemblance to the true and the eternal Forms, and the exclusion of the opposite eternal Forms.

In the argument of the opposites Plato points out that each thing comes to be from its opposite, for those forms that exists an opposite. For example, the larger comes from the smaller, just from the unjust, sleeping from being awake and vice versa. In the same way, the living comes from the dead and the dead from the living. Therefore, the souls of the dead come to life again in this cyclical change; otherwise if everything endowed with life were to die and remain in the state of death, ultimately everything would have to be dead in the universe and nothing alive (Plato Phaedo 71 72).

"In the same way my dear Cebes, if everything that partakes of life were to die and remain in that state and not to come back to life again, would not everything ultimately have to be dead and nothing alive?" (Plato Phaedo 72 d).

The reason why particularly the soul persists after the death of the body, Plato tries to explain with his second argument of recollection. Beginning with the common knowledge, that everything that we can recollect we must have previously learned, he draws attention to the faculty of mind concerning the similarity and equality of the objects. So, he claims, when we perceive the visible objects through the senses, we are in a position to compare these objects and conclude that e.g. this one is similar to that, or equal to the other or larger than this one etc. This means that we possess the knowledge of the Equal itself, Greater itself and Smaller itself, long before our senses allowed us to perceive the visible objects of wood or stone etc. After we have perceived the objects, we are able to compare them and to conclude that, this piece of wood for example is larger than the other one, or equal to this one. This knowledge of the forms of Equality, Smallness and so on, since we possess it from our birth, means that we have acquired it in the past before our birth, for knowledge is a recollection of what we have previously learned.

"Consider, he said, whether this is the case: we say that there is something that is equal. I do not mean a stick equal to a stick, or a stone to a stone, or anything of the kind, but something beyond all these, the Equal itself (auto to ison). Shall we say that this exists or not?

"And do we know what is this? Certainly.

"Where have we acquired this knowledge?" (Plato Phaedo 74 a, b).

In combination now with the first argument, we can conclude, that the human soul coming from a previous death does not lose this knowledge, which remains as a recollection and recovers it by the use of senses. However, we must consider the fact that, when we recall something from the visible things, during this recollection we always realize that what we recall e.g. a person, object etc., is always an inferior copy of the real person, object etc.

"When the recollection is caused by similar things, must one not of necessity also experience this: to consider whether the similarity to that which one recollects is deficient in any respect or complete?" (Plato Phaedo 74 a).

From all the above considerations we can conclude that, our soul after having acquired the knowledge of Equality itself, and while seeing the visible objects, realizes that these objects appear to be the same with the original equal (perfect), however they are always inferior, just like the memories are always inferior to the objects themselves.

"Whenever someone, on seeing something says to himself: 'the thing I am looking at wants to be like something else but falls short and cannot be like the other, since it is inferior copy'." (Plato Phaedo 74 d).

To sum up, as we had possessed the knowledge of the Equal itself before birth, and since after birth we have the ability of perception through the senses, we try to refer these sense-data of equal objects to this Ideal Equal, but we realize that although they are inclined to be equal, are always inferior to that Ideal.

"Then before we began to see and hear or otherwise perceive, we must have possessed knowledge of the Equal itself if we were about to refer our sense perceptions of equal objects to it, and realized that all of them were eager to be like it, but were inferior." (Plato Phaedo 75 b).

This previously mentioned knowledge of the Ideal Forms that the soul has acquired in the intelligible realm of the Good, remains in its memory, and is manifested as a faculty of comparison of all the perceptible things with the ideal and perfect original Forms, which the soul once has learned in the intelligible realm. That particular knowledge does not only concern the knowledge of the Equal, Greater and Smaller, but also the knowledge of the Beautiful itself (kalon), the Good (agathon) itself, the Just (dikaion) and the Pious (osion), which are called intelligible Forms or Ideas. These Forms are the eternal attributes of the Good itself, of the unchangeable and divine Being. Therefore, the recollection (anamnesis) of all these intelligible Forms of the Being is the knowledge of the truth, of what really exists, of what it is (auto o esti). This knowledge can only be recalled through dialectic.

"Therefore, if we had this knowledge, we knew before birth and immediately after not only the Equal, but the Greater and the Smaller and all such things, for our present argument is no more about the Equal than about the Beautiful itself, the Good itself, the Just, the Pious and, as I say about all those things which we mark with the seal of what it is." (Plato Phaedo 75 c, d).

Referring now to the well-known argument that the soul is a harmony that persists, Plato argues that each harmony is composed of elements, which are in the same state, so if the soul were a harmony, would not have existed before its elements. Furthermore, since the wise soul rules over the elements of which it is composed, which means the appetitive and the spirited part, it is impossible to be a harmony, in which as we know, none of its composing elements can rule over the others.

"On the other hand we previously agreed that if the soul were a harmony, it would never be out of tune with the stress and relaxation and the striking of the strings or anything else done to its composing elements, but it would follow and never direct them?" (Plato Phaedo 94 c).

Further to the matter of immortality of the soul, Plato refers to the imperishable Forms and distinguishes them from the other pairs of opposites (polar contraries), mentioned in his first argument. The Forms although have opposite qualities (e.g. warm cold, odd even etc.), since they are polar contradictories, they neither can tolerate the coming to be from one another, nor admit the opposite Form, but when that Form advances upon them, they either perish or withdraw. For example, the odd number three (3) cannot be transformed into the even number two (2), but each one retaining its quality, perishes or gives way. The same as, when the cold snow approaches the hot fire they cannot change the one into hot snow and the other into cold fire.

"Look now. What I want to make clear is this: not only do those opposites not admit each other, but this is also true of those things which, while not being opposite to each other, yet always contain the opposites, and it seems that these do not admit that Form which is opposite to that which is in them; when it approaches them, they either perish or give way." (Plato Phaedo 104 b).

According to this argument of the exclusion of the opposite Forms, since the immortal soul is within a body which is alive, when death comes, the body will change from alive into its opposite dead (polar contrary), while the soul since it is an imperishable Form, will give way to death and will leave safe and indestructible for another place. As the body possesses the quality of mortal can be transformed from alive into its opposite dead. However, as the soul possesses the quality of immortal Form, cannot be transformed into its opposite mortal, but it simply gives way to death and it goes away.

"Then when death comes to man, the mortal part of him dies, it seems but this deathless part goes away safe and indestructible, yielding the place to death." (Plato Phaedo 106 e).

Excerpts are from by book Handbook of Greek Philosophy

Nikolaos Bakalis

back

(59) Anders asked:

I have this question bothering me but I feel the answer might be answerable in quite short terms. If I state that Spinoza is a Monist thinker compared with the dualistic thought of Descartes. Would this be an epistemological difference or an Ontological one?

---

I would say that for each respective thinkers, their epistemologies are also ontology's. Both claimed to have discovered the nature of What Is, by the utilisation of the correct mode of reasoning.

Assuming that Ontology is the nature of what Is, there can only be one correct Ontology accessed by a correspondingly correct epistemology. So either Descartes or Spinoza has utilised the correct epistemology to the exclusion of the other or, they are both wrong. An epistemological difference is also an ontological one: either Spinoza is correct and Descartes is wrong or Descartes is correct and Spinoza wrong.

That a- priori reasoning - the hallmark of Rationalist metaphysics - is not a productive epistemology was highlighted by Immanuel Kant in The Critique of Pure Reason.

Martin Jenkins

back

(60) Lindsay asked:

What Philosopher said 'God is Dead. Superman shall live'?

---

Friedrich Nietzsche [1844-1900]. In The Prologue of Thus Spoke Zarathustra by Nietzsche, Zarathustra states that God is Dead. In descending from the Mountains where he has existed in solitude for ten years, he visits a town. Here, he tells the inhabitants that he brings them the Ubermenschen, that man is something to be overcome and that the Ubermenschen shall be the meaning of the earth. Ubermenschen can be translated into English as Overman, Beyondman and not simply as Superman. The latter conjures up Nazism, Master Race, Blond beast, elitism and so on. The Ubermenschen can be understood as the creative source that re-evaluates existing values and not as a race of herd oppressing supermen. People of tomorrow [perhaps even of today?] will have no recourse to God for the origin of their values and valuations. They arise naturally from people and circumstances articulated by sensitive, creative, active types. That is why present day man is something to be overcome. With recent events concerning cartoons, this is germane.

Martin Jenkins

back

(61) Richard asked:

It may be agreed that much popular philosophical dispute arises because of misuse, ambiguous use, or variant usage of words. The OED provides us with technical and scientific terms that are usually closely defined by the scientist who coined them. Would it not be a service to the English language to have philosophical terms in the OED more closely defined, preferably by philosophers? Such a term as God or gods, is so ambiguous as to seem almost useless. Even a term like racism appears to be so vague, setting aside any legal definitions, that anyone voicing a mere aesthetic preference may be accused of racism. Naturally, definitions may be changed over time as ideas evolve.

---

It is always a service to have words defined and ideas clarified, but I don't see that it is specially a service, either to philosophy or the work of the Oxford University Press, to have them defined by the OED. I think you mistake the OUP for the Academie Francaise. The OED does not define words. Rather, the OED records the senses in use - quite a different matter, as may be seen by the fact that for many important words multiple and variant usages are recorded. In the case of many philosophical words, even this lesser task is a tall order. Why change the project of the OED entirely? It seems to me that the recommended policy would not lead to a gain for Philosophy, but rather to a loss to English of a useful reference work, and it's transformation from a report into a manifesto. It is the job of a mathematician to be clear about his axioms: likewise the philosopher must tend to the clarity of his terms. For things to be otherwise would be to make lexicographers philosophers. What then will the philosophers do, when they need a lexicographer?

David Robjant

back

(62) Petros asked:

The philosopher Ludwig Wittgenstein once stated that there can be no philosophical question asked on the grounds that they are merely linguistic riddles word puzzles that have a specific or set of solutions based on the assumptions and definitions accepted by the person solving the puzzle. If Wittgenstein is correct then shouldn't the title of this web site be changed indeed shouldn't the very existence of philosophical enquiry be challenged?

---

Yes. But he isn't, so it shouldn't be.

David Robjant

back

(63) Ann asked:

Why do they call it a "building"? It looks like they're finished. Why isn't it a "built"?

---

Tricky problem. Possibly philosophical. Not sure. This is etymology, but I think the english and anglo-saxon use of the nordic and germanic 'ing' ending has evolved over the years, so that the use of 'ing' for the transitive is the anomalous innovation, and not the use of 'ing' for a finished article.

David Robjant

back

(64) Michele asked:

Both Blackburn and Arthur casually allude to Plato's dialogue Euthyphro as the locus classicus of the decisive refutation of a religiously based "command morality". The sheer casualness and brevity of their allusion tells you much about how decisive and final that refutation is usually taken to be. How is that supposed to work exactly?

---

I cannot speak for Blackburn etc but I can speak to the question.

Question: is it Good because God Commands it, or does God Command it because it is Good? If the latter, the "command morality" thesis is false, since God's Commanding it because it is Good prohibits it's being Good because he commands it. And if the former (is it Good because God Commands it), then this leaves the question of why God commands it mysterious, and this lacuna may be problematic for various reasons, some simply theological, others logical or grammatical.

Now, some say that we do not need to allow for any insight into God's decision making. His ways are mysterious to mortals, and out lot is to accept them (read your Bible/Koran and obey the word of God!).

However, one problem in theology is that traditionally a large number of things are said about God in addition to the claim that his commands are good, and one can have a hard time fitting these other claims together with the thought that 'good' simply means 'whatever God chooses/commands'.

For example, Christians want to say that God is Good. But how can this be made sense of alongside the claim that 'Good' is wholly defined by what God commands or chooses? That God chooses or commands himself is pretty odd as a sentence in English, and hard to understand as a claim. Who does the choosing? Doesn't the grammar of 'choice' imply some separation between the chooser and the chosen? Well, 'I choose myself' can be made sense of in some contexts, but those contexts are limited, eg 'Of all the people who had applied for the job of export consultant, I, as managing director, chose myself'. But in such cases choosing of oneself is constrained by one's having separate roles (myself as managing director, myself as export consultant), and since God has one role, the role of being God, it doesn't seem obvious that the grammar of choosing or commanding oneself can apply here.

Secondly, on a logical front, it is part of what we mean by 'chose' that it is the action of deciding between courses and things with various merits which we are assessing. I.E., 'good' is logically prior, or at least grammatically prior, to 'choice'. There is an informative a difference, here, between a 'choice' or 'command' and what happens in an arbitrary 'lucky dip'. To command something, as to chose something, is not simply to approve of whatever presents itself. We will return to the question of arbitrariness.

Thirdly, if 'good' means 'whatever God chooses' or 'whatever God commands', there are bound to be those who object that under such a conception of God's authority on earth, it resembles the rule of a human tyrant. A Good God could not command arbitrarily, whence it follows that A Good God must command in the light of the Good, and that God's commands are subordinate to the Good, not the other way around.

Something of this third argument is present in the minds of those Christians who intuit, for instance, that homosexuality does no harm to anyone and is not bad, and argue that for this reason it cannot possibly have been forbidden as a sin by a loving and Good God. Such evidence as may exist to the contrary (EG the writings collected in the bible or the Koran), is then thought to be tainted and contaminated by low human loathing of difference, and not be the true word of God. Now, this revision of the status of holy text in the light of our moral intuitions some Christians will not accept (ie that minority who believe that all of the Bible, including various contradictory passages in it, is authored wholly by God without any addition of human thoughts), and many take it to be the identifying feature of all mainstream Islam that the Koran is not only the simple reporting or setting down of the words (indeed the versifying) of God but also the final testament of God (ie that He cannot [or will not?] at any later stage reveal anything contrary through moral intuition). However, if one takes that view and combines it with the 'command morality' thesis, the two aforesaid difficulties about God's Goodness will arise renewed. God cannot chose or command himself. A Good God could not command or chose the moral rules arbitrarily.

These reflections point to, or already have in them, the (in my view) decisive insight that it is possible to ask, of anything or anyone including God and his command, the question 'Is it/he/she good?'. G.E. Moore thought, and I agree, that this fact alone was sufficient to demonstrate that Good cannot be identified with (or 'naturalised' to) anyone or anything, since if such identification were possible, then there would be some question, such as 'Is God's command Good?' which ought to be literally meaningless: a tautology with a question mark. The fact that this question is not meaningless is enough of itself to show that 'Good' does not simply mean 'God's commands'. If 'it is good' simply means 'it is God's command' the plainly open question 'is God's command Good?' would have to amount to the closed non-question 'is God's command God's command?'. But plainly, when we ask whether something is good, we do not mean to ask 'is God's command God's command?'. Therefore the meaning of 'good' cannot be naturalised to 'God's command', or anything else like 'happiness' or 'pleasure'. This is called the 'Open Question Argument', and something of it was thought, by Moore, to be already present in the atmosphere of Plato's 'Euthyphro dilemma' about whether Zeus commands it because it is Good, or it is Good because Zeus commands it. 'What is the good?' asks Socrates in the 'early' dialogues. Not this, not that. Not, for one, 'whatever zeus commands'.

In my view, for all the aforesaid reasons, the second of the Euthyphro possibilities simply cannot be made sense of. There are some "command morality" text-venerators who are untroubled by this. Well, perhaps we should love the words as they do, but if we are to do so, doesn't that also require loving their grammar equally, including the grammar of 'chose' and 'command'? Words without grammar are gibberish, not divine commands.

David Robjant

back

(65) Richard asked:

In ethics, is it not illogical to begin with assumptions about what is right or wrong? Is it not first necessary to relate all ethical/ social/ political values with one another, and only then derive whatever lesson that pattern provides?

---

If that pattern did provide any useful lesson, that might be a proper avenue of enquiry. As it turns out, however, all you learn from seeing all the mass of contradictory values en masse is that there are masses of contradictory values. And the fact that A contradicts B offers, of itself, no useful insight into whether A is right, or B is right, or whether both are wrong. So the anthropology first approach you recommend isn't going to yield any useful ethical insights.

Seeing that "to begin with assumptions" is, in any case, just exactly what the experimental physicist does, I don't see what's supposed to be so wrong with that approach either.

David Robjant

back

(66) Nicole asked:

These are questions asked by my 10th grade English classes. I know there are a lot of questions listed here, but they would be very excited if you guys could respond to a few, maybe over time. Thank you!

1. Is there a parallel universe?

2. Has human nature changed over time?

3. Why do people have emotions and animals don't?

4. To what extent can morality prevent the destruction of the world at the hands of mankind?

5. What is the meaning of human existence compared to animal existence?

6. If we lived in a perfect world, would there be people tempted to mess it up?

7. Are people naturally good or evil?

8. How will the world end?

9. Does a person's environment decide one's morality or does their personality?

10. Is it really better to have loved and lost than never to have loved at all?

11. If we were always happy, would we really be happy?

12. Is the existence of one state of mind or emotion determined by the existence of it's opposite?

Thank you again. The students are really excited to hear how philosophers would respond to these.

---

1. If it were supposed entirely parallel (i.e. intersecting at no point), then given the meaning of 'universe' (the totality of all that is), accepting the existence of a parallel universe would be tantamount to denying the existence of this one. However, I suspect that when some people (including scientists) speak of 'parallel universes' they mean something else, i.e. not parallel, and not a universe. Something with a relation to our part of the universe more like that of a continent connected to our continent by an isthmus ('wormhole'?), or separated from ours by an ocean (of times, space, or energy). There is also the case of 'possible worlds', which are not to be confused with parallel universes, and are a philosophical device for making sense of counterfactuals ('if I'd missed that bus I would still be in Aberystwyth'), on the mistaken philosophical assumptions a) that meaningful and true utterances must refer successfully to some object (me still in Aberystwyth in a possible world), and b) that counterfactuals must be taken to be true in the same sense and degree of truth that it is true that 2+2=4.

2. This is a lexicographer's problem, since it depends what you mean by 'human', and by 'nature'. There is also some interesting mileage to be got out of differentiating 'human nature' from the philosopher's concern, 'the human condition'. Anyway, I'll lay it down in case anyone doubts it that Plato and Aristotle were human beings, and that there's nothing fundamentally different about us which entitles us to think that their thoughts are irrelevant to homo ipod.

3. What possible reason could one have for asserting that animals don't have emotions?

4. To the extent that, if this destruction is positively prevented (rather than accidentally averted), that action of preventing will be informed by some picture of oughts - minimally, that we ought to prevent the destruction of the world. Whether this counts, for you, as being the same thing as "morality prevent[ing] the destruction of the world" I am unclear. Conceived of as one thing called "morality", the mass of various conflicting ideas about the good is as likely to positively hasten the end of the world as to avert it.

5. This does sound like a nonsense question, but I don't know enough to be sure that there isn't anything you mean by it. Do you mean something like: are we humans entitled to dominion over the earth and all the animals in it? Or: is human life more valuable the animal life? Yes to the latter, in the abstract.

6. No, since that temptation would be, self-evidently, an imperfection.

7. No. However, I am inclined to take the lesson of experience that people can become incorrigibly evil. That's assuming that it is other human beings who have to do the correction.

8. With a whimper, and, hopefully, a momentary flash of selfless love.

9. Since ones personality (character) is ones morality, I am at a loss to understand how the one could be 'decided' by the other (they are not distinct). Environment doesn't 'decide' anything either, or we wouldn't call it the Environment (etymology: the surroundings): no, in that case we couldn't distinguish between the mass of influences and a 'person' that they are influences on. That there are persons (in reality, and in grammar) is enough to establish that surrounding factors do not decide morality, but rather surround it (obviously!). This is not to say that environment has no influence, but just that it makes no sense to talk as if the moral agent (the person) were just was the 'sum of influences'. It is of the grammar of 'influence' that it is an influence on something with a mass and energy of it's own, as it were - we cannot just speak of influences on influences, of being 'the sum of influences'.

10. Better for one's intellectual and spiritual development, possibly. Better for one's happiness and lust for life, possibly not. No one has yet had the opportunity of reporting on an experiment in which they relived their life the other way and were able to compare the results, as if choice entered into the matter of love.

11. Yes. But we might have to use other words for our condition, or none at all, if 'unhappy' were to become extensionless in our experience. Maybe using no words at all would be the necessary complement to the desired result, but although not altogether unimaginable (some monks do it), successfully accomplishing such a life might be fraught with difficulty. It might be dependent on the co-operation of the wordy world of unhappiness and happiness.

12. This is the previous question restated. I have offered that the existence of a name or image for a state of mind or emotion is determined by the existence of it's opposite - but that is not quite the same thing at all.

Twelve questions. What a noble number (better than twenty).

David Robjant

back

(67) Lili asked:

Why does Descartes introduce the evil demon, supremely powerful and clever into the First Meditation? What role is this figure assigned to play?

---

Descartes method of hyperbolical doubt was a search for certainty. By doubting everything he possibly could, whatever was left afterwards would be certain. He invented the demon in order to be able to doubt his strongest beliefs: no matter how convinced he might be of the truth of these beliefs, he could be being deceived by this malevolent demon. But the demon could not convince him that he did not exist, since he had to exist in order to be deceived. His cogito ergo sum (I think therefore I am, or I am conscious therefore I am) was his final certainty: he had to exist in order to be conscious.

Helier Robinson

back

(68) Bob asked:

What's so rational about self-interest? In the prisoners dilemma, one sees the optimal solution, but the logic of self-interest prevents one from reaching it. How is this not a proof by reductio ad absurdum that self-interest is not rational Game theory is a branch of mathematics after all? On the face of it, in order to play this sort of game, I have to accept that the other has desires and fears on a par with my own, and then to disregard them in my own calculations Who started this idea of rational self-interest, and is anyone else questioning it? There's a ton of stuff on game theory and related ethics Rawls, and Binmore for example, but they all seem to start as if the idea itself is unquestionable. Im trying to write something about this, and Im worried I might have got the wrong end of a long stick, and misunderstood something totally obvious help, please!

---

1) Well, "rational". Um... it's actually pretty easy to deconstruct, if you want to put it that way, the term "rational" and show that it's a rather confused one. There's a school of thinking, more or less derived from Hobbes, etc., that holds that we're selfish animals, and so motivated solely by self-interest. You might check out Hobbes, and then Hume's counter-arguments, which, actually, I think, are correct. And then there's the modern, Randian version, which tries to argue that this is desirable... anyway, if you look at some of the modern computer simulations of ethics and of evolution, you find that cooperation generally is better for groups. See below.

2) The problem with the Prisoners' Dilemma is that you're in jail with people who might not want to cooperate, however much it would benefit them... they just don't believe it would. I mean, read the newspapers. So given that, and given the severe penalties to you if they do not cooperate, then what do you do? If you can talk to them, maybe you can work it out, but if you can't... So the reductio is indeed true for the ideal case, when you can talk, but in this case, the best payoff, which is not all that great, is probably to assume the worst of the others. To put it another way, just because it's best, and just because many people cooperate, doesn't mean that those are the ones with whom you're locked up. The Hobbesian solution, however, is to assume, in effect, that you can't talk to anyone, and that even if you do, they'll do their best to lie to you. The Humean rejoinder is, hey, just go to your local tavern and you'll find people actually enjoying company, enjoying doing good things for others, and actively seeking other's liking and approval. I think Hume's correct, in contrast to the assumptions of many of our world leaders... but I'm not sure I'd assume that, in a prison, incommunicado.

3) Hobbes, as I said. At least he was the most overt at it. But there are a lot of people questioning it, and the same is true, now, in the field of economics, which used to assume "rational" individuals. Now they're starting to assume that people aren't particularly rational, i.e., that they act at random, from emotion, and/or not in their best interests, and try to simulate that.

You might look at some of these:

Bergman, R. "Why Be Moral? A Conceptual Model from Developmental Psychology." Human Development 45 (2002): 104Ü24.

Blasi, A. "Bridging Moral Cognition and Moral Action: A Critical Review of the Literature." Psychological Bulletin 88, no. 1 (1980): 1Ü 45.

Boyd, R. "Finite Beings, Finite Goods: The Semantics, Metaphysics and Ethics of Naturalist Consequentialism, Part Ii." Philosophy and Phenomenological Research LXVII, no. 1 (2003): 24-47.

Bucciarelli, M., and P. N. Johnson-Laird. "Naive Deontics: A Theory of Meaning, Representation, and Reasoning." Cognitive Psychology 50 (2005): 159-93.

Edgerton, R. B. Sick Societies: Challenging the Myth of Primitive Harmony. 1st ed. New York: The Free Press, 1992.

Gintis, H., S. Bowles, R. Boyd, and E. Fehr. "Explaining Altruistic Behavior in Humans." Evolution and Human Behavior 24 (2003): 153Ü72.

Hsu, M., M. Bhatt, R. Adolphs, D. Tranel, and C.F. Camerer. "Neural Systems Responding to Degrees of Uncertainty in Human Decision- Making." Science 310 (2005): 1680-83.

Lakoff, G. Moral Politics. 2nd ed. Chicago, IL: University of Chicago Press, 2002.

Nieuwenhuis, S., H.A. Slagter, N.J.A. von Geusau, D.J. Heslenfeld, and C.B. Holroyd. "Knowing Good from Bad: Differential Activation of Human Cortical Areas by Positive and Negative Outcomes." European Journal of Neuroscience 21 (2005): 3161-68.

Wendorf, C.A. "History of American Morality Research, 1894-1932." History of Psychology 4, no. 3 (2001): 272Ü88.

Also:

Alexander, J.M. "Random Boolean Networks and Evolutionary Game Theory." Philosophy of Science 70 (2003): 1289-304.

Doebeli, M., C. Hauert, and T. Killinback. "The Evolutionary Origin of Cooperators and Defectors." Science 306 (2004): 859-62.

Greenspan, P. "Emotional Strategies and Rationality." Ethics 110 (2000): 469Ü87.

„„„. "Emotions, Rationality, and Mind/Body." Philosophy 52, no. Supp (2003): 113Ü25.

Hamilton, I.M., and M. Taborsky. "Contingent Movement and Cooperation Evolve under Generalized Reciprocity." Proceedings of the Royal Society B 272 (2005): 2259-67.

Haselhuhn, M.P., and B.A. Mellers. "Emotions and Cooperation in Economic Games." Cognitive Brain Research 23 (2005): 24-33.

Steven Ravett Brown

back

(69) Alexis asked:

What do you say to a woman that she was your girlfriend for 2 years and you find out that she was using you for getting over a previous boyfriend?

---

The problem of "rebound relationships" isn't studied as much as it should be, in modern psychology. In fact, your (ex) girlfriend probably wasn't aware she was using you, to get over a past relationship. At least, I think that it is what happens in most cases.

If you were together for two years, you should probably take into consideration the good times you had together, and that things didn't work out because she obviously wasn't as committed to the relationship as you were. So fight the urge to accuse her, to be aggressive, because you are only angry at yourself, for being so committed and finding out she was not.

Oh, and try not to find someone just to get over her, or you'll be doing the same thing she did to you. Take some time off, let your emotions cool down and don't rush out for a new girlfriend without being sure you're doing it for the right reasons.

Nuno Hipolito

back

(70) Mary Ann asked:

After eating, do amphibians need to wait an hour before getting OUT of the water?

---

This starts from what is a myth, because you actually take 14 hours to assimilate your meal, lastly by your large intestine (last stop for food you eat) and there are very few cases of death or injury for not waiting before getting into the water. As explained here:

http://www.intelihealth.com/IH/ihtIH/WSIHW000/35320/35323/315779.html?d=dmtHMSContent

Nuno Hipolito

back

(71) Mary Ann asked:

Why do they sterilize needles for lethal injections?

---

The reason stated is the prisoner is eligible for a reprieve until he is dead.

Nuno Hipolito

back

(72) Mary Ann asked:

If you try to fail, and succeed, which have you done?

---

You have had some good luck, obviously.

Nuno Hipolito

back

(73) Natalie asked:

"The explanations of Christianity and Science for the origins of the universe are not compatible." Do you agree?

This is my essay question but i don't really understand it!

I have to summarise what the big issues in this question are!

---

"First of all, you need to take a breath. Jeez.

It's not a hard question Natalie. Just think about it for a second. Everyone knows what the Christians think happened in the beginning of everything.

Pick up your bible (you should always have one at hand), and read the first few pages of what is called the Genesis. Genesis means origin, birth. In it, it is described how God, in six days, created even and earth. This story of creation is only one of several that existed in ancient history, and actually it takes some pointers from other stories, from Babylonia to Ancient Egypt. In a nutshell, God creates the earth in six days, and resting in the seventh. Both the numbers 7 and 3 (3+3=6 days) are important numbers to the cabalistic tradition.

The science part is more complicated. One thing's for sure, physicist don't quite believe that all seven days thing. They actually think the Universe was created from a big bang, from where all matter was made and then stars appeared, planets, and so on... read about this here:

http://www.umich.edu/~gs265/bigbang.htm

You have a start here, so just calm down, and develop these issues, researching them a little bit more.

Nuno Hipolito

back

(74) Nicole asked:

These are questions asked by my 10th grade English classes. I know there are a lot of questions listed here, but they would be very excited if you guys could respond to a few, maybe over time. Thank you!:

1. Is there a parallel universe?

2. Has human nature changed over time?

3. Why do people have emotions and animals don't?

4. To what extent can morality prevent the destruction of the world at the hands of mankind?

5. What is the meaning of human existence compared to animal existence?

6. If we lived in a perfect world, would there be people tempted to mess it up?

7. Are people naturally good or evil?

8. How will the world end?

9. Does a person's environment decide one's morality or does their personality?

10. Is it really better to have loved and lost than never to have loved at all?

11. If we were always happy, would we really be happy?

12. Is the existence of one state of mind or emotion determined by the existence of it's opposite?

Thank you again. The students are really excited to hear how philosophers would respond to these.

---

1. Probably, but it is just a theory right now, just a bunch of numbers put together that created a theory, that has yet to be proven scientifically.

2. Human nature, for me, is all about our feelings, that's the one thing that makes us different, along side with our intelligence. And basic feelings, like love, hate, jealousy, etc, have not changed much over time, I don't think. Of course, we learned to better get in touch with them, having more time for love, arts, business, etc, so it could be seen as a change, but I don't think it is.

3. I think animals have emotions. They're just rawer, unprocessed. We feel and that is an important part of our thought, of our human experience, as for animals, it's a part of survival. Probably because they can't process by intelligence what they are feeling. I had a cat that lived with me for a long time, and when he died, I felt a very human connection from him.

4. It can't. Often acts of inhumanity are impulsive ones, not rational ones. And remember that morals change from country to country. Hitler, for example, thought he was doing the moral thing; it was his view of a moral option.

5. See 3. and 4

6. What is a perfect world? We are restless beings, so probably yes, we would tend to always look for more, even if we were indeed satisfied. Being content is not a big human quality.

7. I believe people are naturally good. You never hear of a baby serial killer.

8. If nothing is done to prevent it, our Sun will go red giant and then white dwarf and engulf all the planets in the solar system. But before that, all life on earth would end because of the increased eat and radiation.

9. One's morality no, because morality is all about making choices. One's personality is influenced by environmental issues, and that conditions our power to make choices, even moral ones.

10. Yes, because love is at the heart of human emotions, and the human experience. That's because love is the epitome of not being selfish, and humans are, as are most animals by nature, selfish beings.

11. No, we would probably be on drugs. Actually, I think everyone was happy in the 70's. And that end very well.

12. Hum... there is a negative for a positive, in most things in Nature, so yes.

Nuno Hipolito

back

(75) Jo Beth asked:

Are the numbers of rich people in countries signs of strength or weakness?

---

More wealth is a clear sign of economical strength. For more millionaires and billionaires to exist, the economy must be very powerful, and that provides more income for a stronger, and richer, middle class and upper-middle class.

Of course, there are the famous downsides to capitalism: the abandonment of the social state, the immorality of greed and material possessions, the lack of moral basis for business decisions, etc... So a capitalist country, albeit a rich one, can be a weak moral country, with weak social links, and prone to be affected by social unrest, possibly even riots, at worst a civil war.

Nuno Hipolito

back

(76) William asked:

I was told in 1986 that the quality improvement initiative that became known worldwide as Total Quality Management was coined by an American Admiral as a result of a conversation with his secretary.

As the story goes, the admiral had just returned to his office after a most frustrating meeting of high ranking Pentagon officials. When he said to his secretary, "We need to find a way to get management to support our quality improvement initiatives. Every program we have tried has failed for one reason or another. Quality Circles failed due to lack of management support. Zero Defects failed because management didn't believe in it and our last effort, Total Quality Control also failed because of management. What do you think we need?" To which, she supposedly answered, "Maybe you need Total Quality Management."

I would like to learn the name of the admiral.

Thank you ever so much.

William M. Ford, Ph.D. K.C./Madison Group

---

I think his name was Frank Kelso.

Read this PDF to learn more:

http://permanent.access.gpo.gov/websites/nduedu/www.ndu.edu/library/ic6/93cs05.pdf

Nuno Hipolito

back

(77) Nicole asked

Has human nature changed over time?

---

Answer: Yes. Human nature has changed remarkably. Homo sapiens evolved some 160,000 years ago an a new species of Hominid. Homo sapiens did not have a human nature, nor the set of controlling instincts that more primitive animals had. Homo sapiens had to invent cultures to take the place of instincts and provide a repertoire of behaviour. By inventing cultures Homo sapiens began the long process of ceasing to be an animal in an environment and becoming a human in a community. For thousands of years Homo sapiens were hunter gatherers, exhibiting very little change that we can detect. Some 40,000 years ago there was the Paleolithic revolution. This revolution was characterized by the development of new tool-making technology, the use of new materials for tools, and the use of tools to make tools.

It was not until the last Millennium BC that humans began to think critically and develop a moral sensibility, as Bruno Snell shows in "The Discovery of Mind" (1953). These two characteristics appear to be connected. There are still human cultures that have not reached this stage. So humans effectively make themselves through their cultures. Cultures are processes of human self-creation, as humans make cultures and cultures, to a significant extent, make the humans of the culture.

Tony Kelly

back

(78) Max asked:

My doctor referred to me as an 'old man'.

How do I know if I am an 'old man' and how am I expected to feel now that I have been classified as such.

Obviously I do not recognize myself as the person the doctor is alluding to.

I am 75 years, in good and vigorous health both physically and mentally, probably more active that when I was at forty.

Am I ready to be called an 'old man?'

Thankyou in anticipation of your considerations.

---

1) Find another doctor. Good grief. What a thing to say. 2) As for how you're "expected" to feel... hey, who's doing the expecting here, you or your doctor? 3) Now, realistically, you're not going to live as long as you've already lived. Human beings don't live, by and large, past 100-120. Ok? But, "old"? "Old" means you've stopped moving forward, mentally and physically. You don't seem to have done that, so you're not "old".

Forget it, find another doctor, and dance on this one's grave in a decade or so.

Steven Ravett Brown

back

(79) Steven asked:

What came first in history, the Catholic Church or Beer?

---

Beer. You think I'm kidding? No way. Beer goes way back in human history.

Steven Ravett Brown

back

(80) Robert asked:

1. Is it ethical to search for illegal copies of intellectual property over the internet on stranger's computers?

2. If it is ethical to search for illegal copies of intellectual property, then is it ethical to erase that data from said computer?

3. Is it ethical to leave computer programs on strangers computers that detect and report back if illegal intellectual property is found?

4. Is it ethical to design into the computer processor itself, a registry that when not set by a proper key prevents the execution of illegal intellectual property?

If the answers to 3. and 4. are different, then why are they different?

---

The answer to 1 is "no" unless you have the proper authority to do so. E.g. if you are a police officer with a legal warrant or if I am a child and you are my parent (of course in the latter case we would hardly be strangers). The reason is exactly the same as the reason why it would not be ethical for me to rifle through the private papers in your desk on the off chance of finding something incriminating there.

So the antecedent of the conditional in question 2 is false.

The answers to 3 and 4 are indeed different ("no" and "yes" respectively insofar as I understand 4). Forget intellectual property. It would not be ethical for me to install, say, "Minesweeper" on your computer without your knowledge and consent. But that does not at all imply that it is unethical for the manufacturers of computers to preinstall it as they often do.

Jimmy Lenman

back

(81) Sarwar asked:

I really have two questions. First, I would like to know what are some of the differences between post-modernism and structuralism. Second, what does Nietzsche mean when he says 'God is Dead'?

---

Structuralism.

Structuralism is contrary to that epistemology that begins with the Subject and how this can know of the Object. Structuralism was / is the attempt to overcome this division and its associated problems [seen in the works of Descartes, Malebranche, Locke, Hume and Berkeley], by citing the identity of the subject and object in the structures of human knowing. Objective structures 'structure' what is perceived providing the 'subject' with meanings and understanding. There is no pre-existing Subject existing external to, prior to and outside the structures of human knowing, only the structures. So, scepticism such as Cartesian doubt is only possible upon the basis of pre-existing knowledge of a world which is subsequently doubted. Some thinkers argue the structures are innate [Chomsky], others that they are acquired by political socialisation [Marx, Althusser], are anthropologically determined [Levi-Strauss], are scientifically-sociologically determined [Foucault] and so on depending on the thinker. Structuralism maintains that meaning; knowledge is identical with those structures. Meaning is presence of the knower with what is known. Knowledge of this knowing is transparent and immediate. Post-Modernism questions this identity, presence and transparency.

Post-Modernism

Jacques Derrida questioned the identity of a text with meaning. Text is far from stable portraying 'traces' which can allow its 'de-construction' to reveal other meanings. Contrary to the claims of structuralism, textual structures are not singularly identical with a singular meaning present to

the reader. De-Construction demonstrated the arbitrariness of Structuralism's and Western Metaphysics' attempt to obtain an identitarian, definitive presence / identity of meaning with text. In other words, 'Logocentrism'as the defining feature of Western Thinking is challenged.

Another 'post-modernist' approach can be found with Jean-Francois Lyotard. In a trajectory which begins with 'The Post-Modern Condition' and ends with 'The Differend', Lyotard examines modes of thought and cognition, which departs from, established universal meta-narratives [can be termed structures] in politics and knowledge. Upon Kant's doctrine of the sublime [from his 'Critique of Practical Judgement'], Lyotard sought to highlight instances of language, communication, knowledge claims, which were other to or outside of existing judgements so escaping subsumption under them. He hoped to demonstrate that the existing meta-narratives were not exhaustive and all encompassing but that language etc were in continuous development. The role of the philosopher is almost like that of an epistemological mediator - s/he seeks to give voice to that which would otherwise be silenced. So this is another example of post-modernity criticising the claims of structuralism to provide a definitive and fixed structure to meanings once and for all. Finally, upon the structuralism of Marxist philosopher Louis Althusser there developed what was termed 'Euro-Communist' thought which led arguably, to a post-modernistic rejection of Marxism and an attempt to chart the territories beyond it. Althusser argued that society was characterised by structures. [see 'For Marx' & 'Reading Capital'] These determine consciousness. An overarching 'structure in dominance' such as the 'economic in the last instance' links structures. The latter qualification of 'last-instance' implies that society is too complex to be reduced to economism as Hegelian Marxism allegedly did for, there as many instances prior to the last. However, upon such complexity came heterogeneity and the departure from anything vaguely Marxist; structures became quasi independent and instances in their own right. Although Althusser certainly had not intended it, his works facilitated thinkers like his former student Michel Foucault taking up 'structures', applying some Nietzschean insights and developing the analyses of 'structures', their creation, enforcement and transgression, in radical, new ways.

Nietzsche's pronouncement that God is Dead

Generally, Nietzsche pronouncement is both an observation that 'God' has been removed from the scene of human knowledge and that the old accounts of knowledge: God, metaphysics, Christianity and Platonism are the history of an error. [see Twilight of the Idols: Reason In Philosophy at al] Nietzsche maintained that there had been a rebellion by the miserable, unhealthy, strata of society against the higher, happier, and more joyful. This rebellion in 'morality' entrenched an epistemological miserabilism colouring the subsequent development of thought. Thought became what was to be termed 'metaphysics'. Metaphysics can be characterised amongst other things by its emphasis on dualism. In other words, what exists, is characterised by dualities. For examples, there is juxtaposed an ultimate realm of reality to the worldly one we live in now, that what exists around us including ourselves owes its existence to a higher origin, that there are ontological structures of reality and unreality of essence and appearance, a morality that says the meek are good - the powerful bad, of intellectual free will caused by an individual soul or ego against bodily passions and so on.

In language, metaphysics structures thought with its categories of essence - attribute, soul and acts: subject and predicate. So we view a 'doer' behind each 'deed' responsible before God, Society, the State, and him/herself for his/her actions. We view an unchanging essence holding the truth of things behind their mutable appearance - perhaps structuralism was one of the most recent manifestations of this approach. The concept of God is the keystone to this edifice of metaphysics which has dominated Western thought for over two thousand years. By announcing the death of God, Nietzsche removes the keystone. And what replaces metaphysics - well, that is up to us!

Martin Jenkins

back

(82) June asked:

Do angels exist?

---

No. Because if they did, then so would plumed serpents; Hindu demons; jackal gods; Thor's goats; Apollo's horses... you get the picture? Once you open the door, everything is in the pot.

Steven Ravett Brown

back

(83) Paul asked:

Thanksgiving day. I am thankful, but to who? My wife thinks that you can be thankful for, but don't necessarily have to be thankful to. I am thinking that if you are to be grateful or give thanks for something then that implies that you are grateful to something or someone. If I was thankful to no-one then I wouldn't be thankful. Do you agree?

---

The way the words work, you can both be right. I thank my lucky stars, or the Lord, or my gracious host. The act of expression seems to demand at least some direction, though it can be more or less focussed. I don't know who or what my lucky stars may be, so in a sense my thanks are undirected, but still I have a sense that they are directed to whatever is appropriate. It didn't rain on my parade; I don't know who to thank, but I know who not to thank, at least.

Gratitude is more a feeling, which might be expressed in thanks or in another way. It comes from the Latin gratus, which is also the root of the word grace and may be related to gratis, meaning freely given without obligation. It seems to me that this feeling of fullness and freedom (from want?) might be expressed by a completely general benevolence that does not exclude George Bush or Osama bin Laden.

The heart of the matter is that to be thankful, to be gracious, is to refrain from claiming all the credit for oneself, a way of life to be encouraged every day, and not just once a year. What is more important than the to or the for is that it is from oneself. This is the secret of happiness.

Bob Macintosh

back

(84) Shirley asked:

I am interested in the Philosophical views that examining the question: Should Science be regulated by ethics? Should the scientist just forge ahead and discover things that may pose great ethical dilemmas? For example, what would most philosophers say about cloning A HUMAN BEING?

---

These are ethical questions, not scientific ones; the word should is a dead giveaway on that. So we are not going to look to science for answers. It is fairly uncontroversial to say that for example some of the experiments conducted on the inmates of concentration camps by the Nazis were immoral, and that certain practices which could be used to answer scientific questions, should not be undertaken, even if the results might be beneficial to humanity. In this sense, science should be regulated by the same ethical considerations that govern the rest of our lives. Likewise, it is incumbent on scientists to consider for whom, and for what purpose, they are working. Better bombs for terrorists no. Better bombs for legally constituted governments possibly. Better explosives for quarry workers quite likely. The problem though, is that these are all the same piece of science. Should Einstein have kept quiet about E=mc2? Could he have known that it would lead to the atomic bomb, and is that out-weighed anyway by the other benefits of his understanding? It is not at all clear how one could reach a judgement on these things even after the event, let alone before one has made the discovery.

It is an unfortunate fact that humanity makes progress in knowledge at an ever-increasing rate, but no progress at all ethically. We do not seem to be less greedy, selfish, hateful and violent than we were, and our increase in knowledge gives us more scope for our immorality. But it does not seem to me that there is anything intrinsically wrong with knowledge itself; it is ethically neutral until we start using it.

Likewise, to take your example, there is nothing intrinsically wrong with human clones identical twins are clones but having said that, it is hard to conceive of a reason for deliberately producing human clones that is not greedy or selfish, or that does not involve the abuse of the persons involved.

What we should do is love and cherish one another, and look after the planet. Scientific knowledge of all kinds would help us to do that better if only we did what we should.

Bob Macintosh

back

(85) Kipyegon asked:

Given individual existence in a state and give the state control over individual, are there instances when an individual is justified to disobey the authority?

---

If you reject the idea of the divine right of kings, emperors, popes, presidents, and such like, then the answer must be yes. If the state is a mafia, if the state is evil, if the state is unlawful, then its laws should not be obeyed. A just person, whose concern is with everyone, rather than his own self-interest, must make a judgement of the particular government in which he finds himself. This government will inevitably be less than perfectly just, but may be the best that can be hoped for. His actions will fall some where along a spectrum that runs from supporting and improving, through efforts to alter and reform, to denunciation and rebellion. I would caution that both ends of the spectrum have great danger. To think that the state is always right leads to tyranny, and to think that it is always wrong, leads to the chaos of constant revolution. It may also be, of course, that a just man has other concerns which he finds more important than government.

Bob Macintosh

back

(86) Daniel asked:

One for the aestheticists here: Why do we enjoy listening to music that makes us feel quasi sadness (it's not real sadness after all because it's not directed at anything)? As a rule, do we not tend to avoid what makes us feel any kind of sadness?

---

When I am sad, I like to listen to music that reflects my mood. Music is evocative, it brings out of me the sadness, and expresses it. Then it is not so much in me and I feel less sad. Even if I don't feel less sad, I feel less alone with my feelings, because the music communicates; the composer, the performer and the listener are sharing the same feeling. I think sadness is a real feeling that everyone has, and music is the best medium for that communication. Perhaps we like to watch horror films for similar reasons to do with fear.

Bob Macintosh

back

(87) Maria asked:

According to Kant, why does the connection between our experiences and our perceptions HAVE to be necessary? (Critique of Pure Reason B219). How does Kant provide examples to support this notion? Would Hume agree or disagree with Kant's notion?

---

This is a particularly opaque passage. I don't think that Kant is saying that there has to be a necessary connection between our experiences and perceptions as such. He's saying that we can only have coherent "experiences" because of our perceptions are necessarily connected with each other into a "manifold" of space and time. If they were not so connected for us, we could not "experience" things. In my understanding of what he is trying to say, our understanding of space and time are preconditions for any perception. Hume simply states that our "impressions" are mirrored by "ideas" in memory or imagination, and that these impressions and ideas can be simple or complex. Complex ones are just multiples of contingently connected simple impressions or ideas. Simple and complex impressions and ideas make up our total "experience". Hume doesn't see the need for this "manifold" at all, for him space and time are merely impressions, or relations between impressions and ideas.

Roger Williams

back

(88) Rebecca asked:

I am writing a dissertation entitled, 'Freewill, autonomy, and mental illness', any suggestions? I have yet to begin it!

---

If a doctor "treats" a mental patient, has the doctor robbed the patient of their autonomy, by forcing them to conform to a behavioural norm? If a being is entirely free to make any choice it wants, does the idea of its "mental illness" or "psychotherapy" make any sense? Why? If doctors can successfully cause behaviour to change, does it make any sense to say that the patient's behaviour was ever "free"? Is there any kind of behavioural change that would rob a patient of their autonomy? How? Can a doctor help us regain our free will, and if so, how did we lose it?

Roger Williams

back

(89) Jone asked:

I have to do my first essay for my philosophy class and the question, "Present the two arguments that induced Hume to deny the strict identity of self or thinking being," also, "Present the argument that induced Hume to conclude that the self is a composition of perceptions." I would be grateful for any help you could give me to orientate my answer.

---

Hume said we have "impressions", which means items of data from our senses, and "ideas", which are copies of "impressions" in our memory or imagination. Everything we know, we build up solely from these. Can we build up any coherent idea of our "self" from this? How do you know that you are the same person you were yesterday, or this day three years ago? Are you the same person? What exactly is it that remains constant over time? What is the thing that perceives your perceptions? What do you know about it? Hume thought there was no certain answer to these questions. Have a look at the 'Treatise of Human Nature', Book I, Part IV, Section VI, and see why. Look at his discussion about the ship that was continuously repaired and rebuilt with new materials: do we have any more certain "identity" than this ship has?

Roger Williams

back

(90) Chi asked:

"My practice, you say, refutes my doubts" ? When Hume says this, what does he mean? How does he reject this objection? Why does he go on to say: "But you mistake the purport of my question"?

---

If he thinks seriously about philosophy, he is only able to be certain about propositions of logic, not about contingent matters of fact. For practical matters, he says he relies on experience and habit, as he does not need such a high degree of certainty. Much of his philosophy, for example, his theory of the passions, of the will, and of moral sentiments, depend on his reliance on experience and habit, and on contingent matters of fact. The purpose of his question, is to ask: "What reason do you have for saying you believe this?", not simply, "What are we going to do next?" In the equivalent passage in the 'Treatise of Human Nature', at the end of Part I, he is much more revealing about his sentiments towards his own philosophy.

Roger Williams

back

(91) Wing asked:

I am doing a research on this question: is it possible that hypnosis and mental illness are left from our bicameral past?

---

This assumes the validity of Julian Jaynes's hypothesis of the "breakdown of the bicameral mind". This is still controversial. Firstly, Jaynes's theory falls foul of Occam's razor: that the causes postulated for a phenomenon should not include elements that are unnecessary for explaining the phenomenon's occurrence. Jaynes has some intriguing observations about how the psychological life of people in ancient civilisations differs from people today, particularly in their relation to the subconscious, based mainly on literary evidence. Although fascinating, these phenomena can be adequately explained by literary conventions or social expectations, and speculations about evolutionary change are unsupported. Secondly, many people have questioned Jaynes's data, both by comparison with literary evidence from other cultures, more examples of practices of other eras, and new research redating many of the texts he was working with. Thirdly, since Jaynes published his work, the tendency has been to try to make psychology more strictly scientific. This means setting up well-formed experiments that can potentially falsify specific predictions. These experiments have shown that the old "split-brain" theories, which Jaynes was relying on, are for too simple. Jaynes's basic hypothesis is probably untestable in practice. However, he does make some statements about child psychology and the development of consciousness that it might be possible to formulate in a way that could be tested by experiment.

Roger Williams

back

(92) Cheryl asked:

What is Kant's ethics of duty? How practical is his philosophy? Are his ideas on ethical duty misplaced or impractical in this modern world? How valuable is his philosophy despite its criticism?

---

Kant's basic moral principles seem very practical: that we should act in a manner that could be a model of a moral law for everyone equally, that we should respect others as ends in themselves, and not as means to an end. It is hard to see how any system that did not incorporate ideas such as these could be called "a system of morality". It is hard to see how they could become "misplaced" or "impractical", unless you were to argue that "morality", as a whole, was impractical. This does not mean that his arguments for his ethics are valid. Ethical principles may exist in some sense, but may not exist as an intelligible moral law for any possible free rational agent, as he claimed.

Roger Williams

back

(93) Coleman asked:

To some, this may be a fairly simple concept, however I am having a difficult time in understanding the concept of moral absolutism versus moral relativism. I would appreciate any comments, examples or theories that would aid me in understanding this concept more fully. If one were to model a society on these concepts how would relativism effect this society and how would absolutism affect this society?

---

Moral absolutism says that there are at least some moral principles that are right in all social conditions. For example: many people say "Women should stay at home with the children," or "Slavery is evil." Moral relativism says that some moral principles, at least, depend on the society and community in which the people live. Therefore, at least some moral principles are only right or wrong for that community, and not for other communities. For example, some say: "Today, it is considered good for mothers to go out and work to help support the family, equally with men." or "It is silly to condemn Benjamin Franklin for owning slaves, as it was the norm in his day." Obviously, one could be a moral absolutist on some questions, and a relativist on others. There is no obvious way in which a moral absolutist society would logically differ from a relativist one. Superficially, one might consider relativists to be more tolerant, but there is no logical reason why this should be so. Both societies could create and change laws in the same way: it is just that the moral absolutists would likely be stronger in their condemnation of other cultures and other eras.

Roger Williams

back

(94) Robbie asked:

I have a few questions to ask. What exactly is philosophy and does it involve Greek mythology? What kinds of philosophy are there? Can you give me some basic information on the origins of Greek mythology?

---

Philosophy is the study of the most basic questions of life or science. If you look at any subject deeply, and start asking, "What is this subject about?" "Do we really know anything about it?" "What can we really say about it?" then you are starting to do some philosophy. The main divisions of philosophy are usually considered to be: 1. Epistemology: this tries to answer the question: What do we know? When can we say we know something? 2. Ontology: this tries to answer the question: What exists? When can we say of something, "It exists!"? 3. Ethics: What should we do or not do? What is morally right and wrong? When can we say that something is morally right or wrong? Some might say that there are other kinds of philosophy. For example, logic looks at the relationship of ideas and statements, and divinity looks at our knowledge of God and the spiritual. No, philosophy has no special relationship to Greek mythology. For that, it would be better to ask on a site devoted to Greek mythology.

Roger Williams

back

(95) Sean asked:

Three questions: From Hume's "concerning liberty and necessity": 1. Does Hume assert that our knowledge of human character and behavior is based on our observation of patterns and repetitions in past experience? 2. In Hume's view, do natural events and human events both exhibit regularity and uniformity? 3. According to Hume, do those human actions for which we praise or blame people exhibit higher uniformity or predictability and therefore are they necessary?

---

1. Yes, but it is based on our observation of patterns and repetitions of all phenomena, not just those directly relating solely to human behaviour. 2. Yes, and in so far as they do not exhibit regularity, they show the effects of "chance". He adds that we attribute the "chance" to our imperfect knowledge of the details of the events, not to a difference in the operation of cause and effect on the events. 3. No. He argues that all actions are in principle equally uniform in their predictability. He says that we threaten to punish wrongdoers in this world or the next, as we think that this will cause their actions to be better. We believe that the threat may cause would-be wrongdoers to change their actions. If we believed the will was entirely free and not subject to the laws of cause and effect, there would be no point in threatening punishment. Hume does not feel that the actions for which we praise or blame people are based on any principles of reason or necessity. He describes them as sentiments arising solely from human conventions. See the 'Treatise of Human Nature', Book III, Part II, Section II, 'Of the origin of justice and property'.

Roger Williams

back

(96) Faris asked:

Why, or why not should one lead the philosophical life?

---

We all lead lives based on some philosophical conclusions. Some have thought only a little about it, and some have thought more. In principle, if you think carefully, you should be able to avoid confusions and inconsistency in your life, and avoid acting at cross-purposes with yourself. Then again, some people just like thinking about this kind of thing.

Roger Williams

back

(97) Erdal asked:

Is there any relationship between economic history and the history of economic thought? if not, explain the alternative path to be taken in order for this discipline to make any sense.

---

It's not clear what discipline you mean by "this discipline", nor what this has to do with philosophy as such. Economic history is the history of what people did in each era: what they made and traded, what things they owned, what they used as money, and so on. The history of economic thought is the history of what economists in each era have said are the general laws that determine economic activity. There might not be any relation between the two, depending on how accurate any economists have ever been. In practice, most economists have talked about what they perceive as the main problems of their own era. Mercantilists talked about how to increase state income relative to one's rivals to produce the most powerful army and navy. Classical economists talked about removing restrictions on trade, so as to promote the maximum growth of global economic power. Marxists talked about the gap between owners and non-owners in capitalist states, and then later about how to create the fastest growth in military output in centrally-planned states. Van Mises and Hayek talked about the dangers of centrally-planned economies. Keynes talked about how to create full employment.

Roger Williams

back

(98) Janette asked:

Hi, I'm in a 120 phil. course at a university, and I'm having some difficulty grasping the concepts. One of them being from Voltaire's Candide. At the end, Candide says, "we must go work in the garden..." What exactly does that mean metaphorically? If you can help me with this, it would be well appreciated.

---

I have always assumed this was a literary reference to the Athenian philosopher Epicurus, who retired to a private garden, and lived off its produce. Epicurus, much like Voltaire, was a materialist but deist philosopher. He argued that pleasure was the only good. To obtain the maximum pleasure, he advocated retiring from public affairs and living a tranquil private life. Although Voltaire may have been advocating Epicurus's principles here, Voltaire's actual life was not at all similar.

Roger Williams

back

(99) Noah asked:

What is a concise definition of positive and negative freedom?

---

Negative freedom means that there is nothing stopping you. Positive freedom means that you are able to choose whatever you wish. Typically the kind of factors that can make a difference are, for example, addictions, economic opportunities, physical abilities, knowledge of other choices, temptation, suggestions, prior experience, even free will. Different philosophers seem to draw the line between the two in subtly different places. Some do not recognise a meaningful distinction. In the 'Groundwork to a Metaphysic of Morals', Kant said the freedom of the will could be defined in a negative sense, as a will independent of external factors, and defined in a positive sense, as a will determined by its own internal laws. He seems to have understood this as two ways of defining the same thing.

Roger Williams

back

(100) Leon asked:

I have been reading David Hume on causation and cause and effect, but I am wondering if there were any philosophers who attempted to destroy the cause and effect relationship altogether or said that there was no distinction between the two (or we could never tell the difference)? And how was it done?

---

Most people would argue that Hume has destroyed the cause and effect relationship. Hume recognizes that we observe a 'constant conjunction', but argues that we cannot prove any 'necessary connexion'. Bertrand Russell in his 'Problems of Philosophy' gave a memorable example of a chicken that develops the law of nature that the sunrise causes the farmer to appear with food, until one morning the farmer wrings its neck instead. Russell's serious point is that we can never prove any law by induction. He argues that the "law" of cause and effect only superficially appears to hold good because there are many examples of it. He tentatively suggests that we apply it understanding this limitation.

The nuclear physicist, Werner Heisenberg, claimed that the "law" of cause and effect does not apply in an entirely deterministic manner for subatomic particles. Instead, he argued that even given a defined "cause", we can only state what the probabilities of any particular outcome might be. We cannot predict a definite "effect" with certainty. Other physicists, such as Einstein and Schroedinger, thought that this was an impossible state of affairs, largely on philosophical grounds. The majority of scientists working in the field today apparently feel it is unlikely that we will ever find a set of deterministic "causes" and "effects" underlying Heisenberg's probabilities. Others argue that a measurable probability can only occur in a deterministic system about which we hold imperfect information. Physicists, of either the quantum or Newtonian variety, do distinguish between cause and effect: the effect has a lower level of energy difference, or less information, or more randomness, depending on which measure you use.

Roger Williams

back

(101) Denise asked:

What are the differences and the similarities between Christianity (a religious ideology) and Marxism (a political ideology)?

---

Both have a messianic ideology, leading to a perfect state containing the perfect human. Both look forward to this messianic event. Both led historically to totalitarian states - the late Roman Empire, medieval states, compared to the USSR. This messianic promise is present in the Gospels, but stronger in Paul and in the Book of Revelation. Similar messianic promises are found in Marx, but are far more prevalent in Lenin, in particular in "State and Revolution".

Both Christianity and Marxism are commonly attended with a conversion experience, which involves induction into a closed group that shares the ideology. Both have commonly censored scientific theory they see as dangerous: Gallileo vs. Tycho Brahe, Darwin vs. Lysenko, etc. Both have resulted in massive purges of dissenters, heretics, or other undesirables: usually with accusations of physically-impossible crimes. Both created extra-state organisations - church/party - that oversee the state, and special investigative bodies such as the Inquisitions and NKVD, aimed at controlling the people, state, and church/party according to the ideology. The differences are many, and significant. Christianity provides a specific detailed moral framework for individual action. Marx never did, and most Marxists have not developed one worth mentioning. Christianity sees the messiah as coming from outside the material world, whereas Marxists see it coming from the inexorable development of the material world, through scientifically-verifiable laws of history. Marxists typically do not accept that there is anything "outside" the material world. Christians often claim to have special spiritual powers, to perform miracles, whereas Marxists only claim to understand the scientific laws underpinning historical events. More can be said on both sides, but that is enough to start. It is probably valid to speculate that the similarities are largely due to Europeans with a Christian background (Marx was born and raised in a Christian, not a Jewish, family) subconsciously recreating the states and organisations that founded their culture.

Roger Williams

back

(102) Carrie asked:

What is the scientific view of the world and compare it to the humanistic view of the world.

---

It is difficult to say if there is a "scientific view of the world". Science is a method for analysing the answers to certain questions. It attempts to develop theories to explain the results of experiments, as described by people such as Francis Bacon, Karl Popper, and Thomas Kuhn. As it only answers certain questions, it does not necessarily involve a complete "view of the world". Also, if you are strict about the method, your view of the world is liable to change with the results of new experiments. "Humanist" is a much more vaguely-defined term. It can cover Petrarch and other Renaissance writers who wished to supplement their study of Christian divinity with the knowledge of the "human" works of ancient Greece and Rome. It can cover Enlightenment thinkers, as well as a narrower set of later non-religious moral philosophies. What they all have in common is that they seek the answer to at least some questions from non-theological sources, in human inquiry. A few people today, mainly Christian fundamentalists in the USA, use the term more narrowly to describe atheists or agnostic philosophies. If this question were to be posed by a religious fundamentalist, I might suspect that they wish to imply that "humanists", by this most narrow definition, assume that there is no God, whereas science is neutral on the question. If it were to be posed by a post-modernist, I might suspect they wish to imply that science is fundamentally amoral, whereas "humanists", by a wider definition, believe in a clear right and wrong. If the question has been posed to you: I would advise you to clarify what the questioner has in mind. If you wish to be Bolshie, and the questioner is clever, you can attack the question for its vagueness, which is what I probably would do.

Roger Williams

back

(103) Royce asked:

Economic theory as accepted in the West assumes the Adam Smith "invisible hand" as fundamental. It is a kind of "deus ex machina". . .nicely permits explanation of many phenomena. At issue is the question of the 'nature of man' ? Hume, vs. Rousseau vs. Descartes etc. Is 'man' basically "self seeking, self aggrandizing and unalterably selfish"? Adam Smith and the market economies count on it for models to lend predictive reliability, apparently TRUE. What do sociobiology, social psychology and moral philosophy tell Adam Smith, Ricardo, Marx and other economic theorists? (See for example Francis Fukuyama.)

---

Although Adam Smith did coin the phrase "invisible hand" when talking about the operations of the market, his view of human nature is not so simple. In particular, if you look at his 'Theory of Moral Sentiments', Smith, like Hume, felt that people had a "moral sentiment" that affected their actions, and made them act in ways that were not solely self-interested in the material sense. Sociobiologists tend to see genes as objects that "aim" to maximise their own survival chances, although some see room for altruism arising in certain contexts for certain creatures within this model. Some social psychology experiments, for example those of Robert H. Frank, suggest that humans in fact do have a "moral sentiment" that over-rides self-interest on many occasions. Frank has some interesting arguments that try to reconcile his findings with sociobiological assumptions. Moral philosophers come in many varieties, but they normally try to develop arguments that tell people how they should act, not just how they do act. I don't know of any moral philosophers who argue that humans should be "self-seeking, self-aggrandizing and unalterably selfish".

Roger Williams

back

(104) Frank asked:

Does Hume's rugged empiricism allow him to say that there is a self? Can he say "I have perceptions" or only "Perceptions occur"? Can one really consider there to be a Humean "self", definable as a "bundle" or "series" of perceptions? Doesn't setting off a particular collection of perceptions require a separate existence serving as a standard, not itself a member of the set, and not itself a perception, to which each member of the set relates in some way? Can a set define itself? Can there be a Humean "self" at all? Did the guy really think he existed? Did he put his self in a pre-Cogito pickle?

---

Yes. Or no, as the case may be.

I think that Hume answers the basic import of your questions when he says: "When any opinion leads us into absurdities, 'tis certainly false; but 'tis not certain an opinion is false, because 'tis of dangerous consequences." (Treatise Bk.2, Pt.3, Sec.2) The idea of a "self" cannot supported with any certainty. Your description of a set defining itself describes his reasons for saying that quite nicely. However, Hume was perfectly capable of living life on the basis that, for example, cause and effect seemed to hold, and his perceptions were stable enough to permit him to continue. He was even capable of continuing to carry on philosophy on that basis. You might want to compare his argument to Augustine's 'Against the Academicians', where Augustine points out that you can put scepticism on one side and carry on by looking at the world and saying you will "act as if it were so." The difference between Hume and Augustine however, is that Hume continues to make a careful distinction between the propositions of logic (relations of ideas) that you can know for certain, and matters of fact that are contingent and of less certainty. Hume argues that other philosophical systems attempt to demand certainty were it cannot be obtained. The consequences of Hume's system may or may not be dangerous, but, he says, the other systems are clearly false.

Roger Williams

back

(105) Eddie asked:

I have not had much experience in the English language as I was brought up in South America and am now going to college in N. America and having difficulty understanding the text and some of the "larger words." My question is as follows: Can someone please explain Kant's Metaphysic of Morals to me in plain English?

---

If anyone ever pretends to you, Eddie, that they are not having difficulty understanding Kant, in any language, you can be certain that they haven't tried to read him yet. Kant is difficult to understand. Don't worry, you've made a good start. Keep going.

Roger Williams

back

(106) Brian asked:

I am interested in creating a program of study in something loosely resembling philosophy of religion. I enjoy reading the works of Nietzsche and Schopenhauer and Kant, etc. I don't want the program to emphasize so much the mental conundrums of "Does God exist?" But more an emphasis on what it is that people are looking or hoping for in "God". Also how the concept of God has changed in the modern scientific era. Is something like this possible?

---

It sounds very interesting. Good luck. If you are not familiar with Karen Armstrong's 'History of God', it might give you further ideas of other directions to look.

Roger Williams

back

(107) Gregory asked:

What the differences of the world of appearances and world of forms?

What is the relationship between the world of forms and world of appearances?

---

As far as I can understand, the question concerns the relationship between the intelligible Forms of Plato and the perceptible things of the visible realm. According to the earlier Dialogues of Plato (Phaedo, Symposium, Republic etc.) the Forms are patterns set in nature and all the things get a share of the Forms by likeness, as being modeled on them, e.g. the Forms of man, horse etc., as well as the Forms of Largeness, Sameness, Difference etc. The Forms are immaterial, unchanging, eternal and abstract particular entities, and different from their copies, namely the perceptible things, which are changeable, material and mortal.

However, Plato in the Parmenides (130a-135d) criticizes his own conception of the Forms with the Third Man Argument (TMA) and proves the inconsistency of his paradigmatism (relationship of model and copy) for some particular Forms, just like Largeness, Difference etc. Later on, for the mature Plato in the Sophist and the Statesman the model is a mundane example (e.g. angler, weaver). Apart from that, he distinguishes five Great Kinds (forms), namely Rest, Change, Being, Sameness and Difference. Although these five Forms are distinct, some of them associate or blend with all the others (Being, Difference), and some of them are incompatible with each other (Rest, Change). As well as some are themselves by themselves (auta kath hauta), just like Being, while some are in relation to other things (pros alla), just like Difference or Largeness. These five Great Kinds hold everything together, make them capable of blending and are the causes of division (Plato, Sophist 250-260).

Nikolaos Bakalis

back

(108) Rob asked:

Why should I go to my midterms next week if the sun is going to die out in a billion years and end everything I've accomplished on earth? Or damn black holes sucking the meaning out of my existence.

What to do...?

---

Um... I'd say it was a bit more likely that the disappearance of our culture, in, say, a few hundred years, or that the languages we employ now will become extinct in, say a few thousand, will render our accomplishments mute. Wouldn't you? But here's the thing. Human interactions are chaotic, in the technical sense... that is, complex enough that any particular event has intrinsically unpredictable consequences. So it may be that the interactions we have, whether artistic, scientific, or purely casually social, will have profound, or no, implications moving down through time, through their effect on people. And it is probably the case that the more interactions, and the more profound they are, the more likely that there will be such long-term effects... that seems to be the historical lesson, right? And so, if you want to increase the likelihood of that, then try for it. As for the sun, galaxy, etc... yes, well... if in a few billion years we aren't at the point where we can avoid such minor matters (yes, minor, and if you don't believe me go here:

http://hubblesite.org/newscenter/newsdesk/archive/releases/image_category/cosmology/distant%20galaxies/

and look around) then hey we probably would never have mattered at all anyway. Go take your midterms.

Steven Ravett Brown